You are on page 1of 58

RESPUESTAS DE LA XXIX OLIMPIADA ESPAOLA

1.- En una reunin hay 201 personas de 5 nacionalidades diferentes. Se sabe que, en cada grupo de 6, al menos dos tienen la misma edad. Demostrar que hay al menos 5 personas del mismo pas, de la misma edad y del mismo sexo. Solucin (M Gaspar Alonso-Vega) Si en cada grupo de 6 personas, 2 son de la misma edad, slo puede haber 5 edades diferentes, ya que, si hubiese 6 edades diferentes, eligiendo una persona de cada edad tendramos 6 personas de edades distintas contra la hiptesis. Como 200 = 2 100 + 1 al menos hay 101 personas del mismo sexo. 101 = 5 20 + 1 al menos hay 21 personas de la misma edad y sexo. 21 = 4 5 + 1 al menos hay 5 personas de la misma nacionalidad, edad y sexo. 2.- Escrito el tringulo aritmtico: 0 1 1 4 ............. 1991 1992 1993 3 5 7 ............................ 3983 3985 4 8 12 ........................................ 7968 ......................................................................... 2 3

donde cada nmero es la suma de los dos que tiene encima (cada fila tiene un nmero menos y en la ltima slo hay un nmero). Razonar que el ltimo nmero es mltiplo de 1993. Solucin. Si representamos los elementos de la primera fila por a0, a1, a2, ........ los elementos de la segunda sern: a0 + a1, a1 + a2, a2 + a3, .............. los de la tercera sern : a0 + 2a1 + a1, a1 + 2a2 + a3, .............. para la cuarta : a0 + 3a1 + 3a1 + a1, a1 + 3a2 + 3a3 + a4,............ Supongamos que los dos primeros elementos bp,0 y bp,1 de la fila p-sima son:
p 1 p 1 p 1 b p ,0 = ; a 0 + a 1 +.......+ a 0 1 p 1 p1 p 1 p 1 p 1 b p ,1 = a 1 + a 2 +.......+ a 0 1 p 1 p

entonces, el primer elemento de la fila siguiente ser :


p p p b p+1,0 = a 0 + a 1 +.......+ a p 0 1 p

(*)

en nuestro caso la primera fila tiene 1994 elementos, la segunda 1993, ... y la ltima corresponde a p + 1 = 1994 y su nico elemento ser
1993 1993 1993 b1994 = 0+ 1+.......+ 1993 0 1 1993

1993 Al ser 1993 primo, es mltiplo de 1993 para todo k menor que 1993 y por tanto b1993 es k mltiplo de 1993.

3.- Justificar razonadamente que, en cualquier tringulo, el dimetro de la circunferencia inscrita no es mayor que el radio de la circunferencia circunscrita. Solucin (F. Bellot) La desigualdad propuesta, R - 2r 0 es una consecuencia del teorema de Euler. Si I, O son el incentro y el circuncentro de un tringulo, r y R los radios de las circunferencias inscrita y circunscritas, se verifica: IO2 = R2 - 2Rr. Entonces IO2 = R (R - 2r) 0 R - 2r 0. 4.- Demostrar que para todo nmero primo p distinto de 2 y de 5, existen infinitos mltiplos de p de la forma 1111......1 (escrito slo con unos). Solucin (Alvaro Begu Aguado) Veamos primero que p tiene infinitos mltiplos de la forma 999...9. Consideremos la sucesin: 9, 99, 999, ......,999...9 (el ltimo tiene n nueves). Entonces se tiene: 9 = 10 - 1; 99 = 102 - 1; 999 = 103 - 1;.......999..9 = 10n - 1 en la sucesin hay infinitos trminos de la forma 10p-1 - 1 con p 2, p 5 y p primo. Puesto que, por el teorema de Fermat: 10p-1 - 1 1 (md p) si p 2, p 5 la afirmacin queda demostrada. Finalmente 999...9 = 9 111...1 entonces si p es primo con 9 (p 3), p divide al producto, es primo con 9 luego divide a 111...1. Queda el caso p = 3 que es evidente ya que los infinitos nmeros: 111; 111111, .......... son mltiplos de tres. 5.- Se dan 16 puntos formando una cuadrcula como en la figura:

A
De ellos se han destacado A y D. Se pide fijar de todos los modos posibles otros dos puntos B y C con la condicin de que las seis distancias determinadas por los cuatro puntos sean distintas. En ese conjunto de cuaternas, estudiar: a) Cuntas figuras de 4 puntos existen con las condiciones del enunciado. b) Cuntas de ellas son geomtricamente distintas, es decir, no deducibles unas de otras por transformaciones de igualdad. c) Si cada punto se designa por un par de enteros (Xi, Yi), razonar que la suma: |Xi - Xj| + | Yi - Yj| extendida a los seis pares AB, AC, AD, BC, BD, CD es constante.

Solucin

El problema admite dos ejes de simetra coincidentes con las diagonales del cuadrado. Clasificaremos las soluciones posibles por la posicin del punto B respecto del vrtice A. Usaremos coordenadas enteras con origen en A. Las tres posiciones fundamentales (no deducibles unas de otras por las simetras anteriores) son aquellas en las que B est en los puntos de coordenadas (0,1); (0,2) y (1,1) para cada una de ellas dibujamos us esquema con las posibles posiciones del punto C. Las posiciones prohibidas se dibujan en negro, la posicin de B en gris y las de Ci en blanco. Un criterio general para prohibir ubicaciones es localizar aquellos puntos que estan en la mediatriz de dos puntos ya situados. Como A y D son dados y fijos, la diagonal principal siempre contiene puntos prohibidos
C1 C2 B A C4 C3 A C3 A D B C1 D C2 B C1 D C1

El esquema de la izquierda contiene 4 posiciones originales y cada una de ellas genera otras cuatro por aplicacin de las dos simetras, en total 16. El esquema del centro contiene 3 posiciones originales y cada una de ellas genera otras cuatro por aplicacin de las dos simetras, en total 12. El esquema de la derecha contiene 1 posicin original que genera otras cuatro por aplicacin de las dos simetras, en total 4. Por tanto existen 32 posiciones posibles y 8 originales, esto contesta a los apartados a) y b). Para el apartado c) hay que suponer que los enteros asignados a cada punto son sus coordenadas en un origen cualquiera, nosotros supondremos que el origen est en A con lo que las coordenadas de A son (0,0) y las de D(3,0). los seis sumandos corresponden a las parajas AB, AC, AD, BC, BD y CD. El correspondiente a AD es constante y vale 3+3 = 6. Los correspondientes a AB y BD valen en conjunto siempre 6 ya que A est en fila inferior y columna izquierda y D en la fila superior y columna derecha. Por el mismo motivo los sumandos correspondientes a AC y CD valen entre los dos siempre 6. Slo queda el sumando |Xi - Xj| + | Yi - Yj| correspondiente a BC que por simple comprobacin en todos los casos originales vale siempre 3. La suma completa es entonces constante y vale 6 + 6 + 6 + 3 = 21.

6.- Una mquina de juego de un casino tiene una pantalla en la que se ofrece un esquema como el de la figura. Para comenzar el juego aparece una bola en el punto S. A cada impulso que recibe del jugador, esa bola se mueve hasta una de las letras inmediatas con la misma probabilidad para cada una de ellas. La partida termina al ocurrir el primero de los dos hechos C siguientes: a) La bola vuelve a S y entonces el jugador pierde. b) La bola llega a G y entonces el jugador gana. A B

Se pide la probabilidad de que el jugador gane y la duracin media de las partidas.

Solucin (F. Bellot)

Podemos representar el desarrollo del juego mediante un diagrama en rbol:


S 1 1/3 A 1/3 S 1/2 C D B D G C D S A C D C 1/3

A S C

B D G

La probabilidad de que el juego tenga longitud 2 es

1 3

1 1 1 2 La probabilidad de que el juego tenga longitud 4 es : 2 2 = 2 3 2 3 3

1 1 1 1 22 La probabilidad de que el juego tenga longitud 6 es : 2 2 2 = 3 , etc, en general 3 2 3 3 3

2 n1 la probabilidad de que el juego tenga longitud 2n es: n 3 Entonces, la duracin media M de un juego es la suma de cada longitud por la probabilidad respectiva : n 2 n1 2 M = n 2 n = n n =1 3 n=1 3 serie aritmtico-geomtrica que se suma por el mismo mtodo que la geomtrica:
2 2 M 2 3 = 2 M = 2 3= 6 M M = = + 2 3 3 3 1 3
2

La probabilidad P de ganar ser la suma de las probabilidades de ganar en 4 pasos ms la de que gane en 6 pasos ...etc.: P= 1 2 22 1 2 + 3 + 4 +....= 3 3 3 3

RESPUESTAS DE LA XXX OLIMPIADA ESPAOLA

1.- Demostrar que si entre los infinitos trminos de una progresin aritmtica de nmeros enteros positivos hay un cuadrado perfecto, entonces infinitos trminos de la progresin son cuadrados perfectos. Solucin Bastar probar que a partir de un cuadrado perfecto podemos construir otro. Sea la progresin: a2, a2 + d, a2 + 2d, ......,a2 + kd......

Como (a + d)2 = a2 + 2ad + d2 = a2 + (2a + d)d, basta tomar k = 2a + d para obtener otro cuadrado en la progresin.
2.- Sea OXYZ un triedro trirectngulo de vrtice O y aristas X, Y, Z. Sobre la arista Z se toma un punto fijo C, tal que OC = c. Sobre X e Y se toman respectivamente dos puntos variables P y Q de modo que la suma OP + OQ sea una constante dada k. Para cada par de puntos P y Q, los cuatro puntos O, C, P, Q estn en una esfera, cuyo centro W se proyecta sobre el plano OXY. Razonar cul es el lugar geomtrico de esa proyeccin. Razonar tambin cul es el lugar geomtrico de W.
Z Solucin En la figura se muestran con trazo discontnuo las circunferencias que resultan de intersecar la esfera con los planos coordenados. Las proyecciones del centro C W de la esfera sobre estos planos coinciden con los G W centros de estas circunferencias (denotados F, G y H H O Q en la figura) y al ser el triedro trirectngulo, F, G y H F P estan en los puntos medio de los segmentos PQ, QC y CP que son dimetros de sus circunferencias. X Parametrizando con la distancia OP= tenemos trivialmente en la referencia OXYZ la siguientes coordenadas: P(,0,0); Q(0,k-,0); C(0,0,c); k k c k c c F , ,0 ; G 0, , ; H ,0, ; W , , 2 2 2 2 2 2 2 2 2

El lugar de F es la recta x + y =

k del plano XOY. El lugar de W es una recta paralela a la anterior 2

c situada en el plano z = , ms concretamente es la interseccin de los planos: 2 k x + y = 2 c z= 2

3.- Una oficina de Turismo va a realizar una encuesta sobre nmero de das soleados y nmero de das lluviosos que se dan en el ao. Para ello recurre a seis regiones que le transmiten los datos de la siguiente tabla:

Regin A B C D E F

Soleados o lluviosos 336 321 335 343 329 330

Inclasificables 29 44 30 22 36 35

La persona encargada de la encuesta no es imparcial y tiene esos datos ms detallados. Se da cuenta de que, prescindiendo de una de las regiones, la observacin da un nmero de das lluviosos que es la tercera parte del de das soleados. Razonar cul es la regin de la que prescindir.
Solucin Al suprimir una regin, la suma de das soleados o lluviosos de las restantes ha de ser mltiplo de 4. Esta suma vale para las seis regiones 1994 que dividido entre 4 da resto 2. El nico dato de esta columna que da resto 2 al dividirlo entre 4 es 330 correspondiente a la regin F. Suprimiendo esta regin quedan entre las cinco restantes 416 das lluviosos y 3416 = 1248 das soleados. 4.- El ngulo A del tringulo issceles ABC mide 2/5 de recto, siendo iguales sus ngulos B y C. La bisectriz de su ngulo C corta al lado opuesto en el punto D. Calcular las medidas de los ngulos del tringulo BCD. Expresar la medida a del lado BC en funcin de la medida b del lado AC, sin que en la expresin aparezcan razones trigonomtricas. Solucin Con los datos del enunciado tenemos: en el tringulo ABC BAC = 36; ABC = ACB = 72 en el tringulo CBD BCD = 36; CDB = BDC = 72 en el tringulo ADC DAC = ACD = 72; ADC = 108 por tanto BCD y ADC son issceles y ademas BCD es semejante al ABC. Para los lados se tiene: DC = AD = a; BD = b - a. Expresando la proporcionalidad derivada de la semejanza anterior:
b-a

D a

ba a a a = a 2 = b 2 ab a 2 + ab b 2 = 0 + 1 = 0 b b a b
2

y resolviendo queda

( 5 1) b a 5 1 es decir a es la seccin urea de b. = a= b 2 2 5.- Con 21 fichas de damas, unas blancas y otras negras, se forma un rectngulo de 3x7. Demostrar que siempre hay cuatro fichas del mismo color situadas en los vrtices de un rectngulo.
Solucin

Dispondremos el tablero en posicin vertical, es decir, con 7 filas y 3 1 columnas. Asignaremos el color blanco a la cifra 0 y el negro a la cifra 1. 2 De este modo cada fila representa un nmero escrito en base 2. En primer lugar es fcil ver que si en una fila se colocan todas las fichas 3 del mismo color, por ejemplo el negro, necesariamente habr un 4 rectngulo ya que no podemos colocar en ninguna fila dos fichas negras y 5 slo podemos llenar un mximo de 5 filas en total sin formar rectngulo. Por otra parte si dos nmeros son iguales sus filas forman rectngulo, 6 luego todas las filas han de representar nmeros distintos. Por la consideracin anterior hemos de excluir los nmeros 000 y 111. Con tres cifras en base dos existen 23 = 8 nmeros distintos, quitando los anteriores quedan 6 para 7 filas por lo que necesariamente hemos de repetir y formar rectngulo. El problema tendra solucin en un tablero de 3x6 tal como se muestra en la figura.
6.- Un polgono convexo de n lados se descompone en m tringulos, con los interiores disjuntos, de modo que cada lado de esos m tringulos lo es tambin de otro tringulo contguo o del polgono dado. Probar que m + n es par. Conocidos n y m hallar el nmero de lados distintos que quedan en el interior del polgono y el nmero de vrtices distintos que quedan en ese interior. Solucin Como hay m tringulos, hay 3m lados; de ellos 3m - n son interiores, y como lado interior pertenece 3m n lados interiores distintos. En particular 3m - n es par, luego m y n a dos tringulos, hay 2 tienen la misma paridad y m + n es par. Supongamos que el nmero de vrtices v slo depende de m y n. Razonemos por induccin sobre v. Si no hay ningn vrtice interior (v = 0), uniendo un vrtice del polgono con los otros, se divide en n - 2 = n + 2v - 2 tringulos. Supongamos que hay v vrtices interiores y n + 2v - 2 tringulos. Al aadir un vrtice hay dos posibilidades: a) El vrtice est en el interior de un tringulo, entonces, para que se cumplan las condiciones del enunciado, debe unirse a cada uno de los tres vrtices del tringulo que se divide en tres y el nmero de tringulos ahora es: n + 2v - 2 + 2 = n + 2(v + 1) - 2. b) El vrtice est en un lado, entonces hay que unirlo con el vrtice opuesto de cada uno de los dos tringulos que comparten ese lado, cada tringulo se descompone en dos y el nmero de tringulos es ahora: n + 2v - 2 + 2 = n + 2(v + 1) - 2. En conclusin: m n+ 2 m = n + 2v - 2 v = 2

RESPUESTAS DE LA XXXI OLIMPIADA ESPAOLA

1.- Se consideran conjuntos A de cien nmeros naturales distintos, que tengan la propiedad de que si a, b y c son elementos cualesquiera de A (iguales o distintos), existe un tringulo no obtusngulo cuyos lados miden a, b y c unidades. Se denomina S(A) a la suma de los permetros considerados en la definicin de A. Calcula el valor mnimo de S(A). Solucin: Si n es el menor de los elementos de A y m el mayor, al tener A cien elementos distintos, ser m n + 99. Para que el tringulo issceles de lados n, n, m sea no obtusngulo debe ocurrir que m2 2 n2 ; si m es lo menor posible, m = n + 99 deber ser (n + 99)2 2n2 , o sea: n 2 - 198n - 99 2 0 n 99 + 99 2 + 99 2 n 99( 1 + 2 ) n 240 . Si n < 240, es seguro que el conjunto no cumple la condicin del enunciado pues m2 (n+99)2 2n2 y el tringulo de lados n, n, m no puede ser no obtusgulo. En particular la condicin se cumple para el conjunto: A = {240, 241, 242, ...., 339} Cualquier otro conjunto que cumpla la condicin, tendr sus elementos respectivamente iguales o mayores que los de ste. Este es, por tanto el que da lugar al mnimo S(A). El nmero de tringulos que debe considerarse es el de variaciones ternarias con repeticin de los elementos de A, que es 1003 = 1000000, con lo que el nmero de lados en total ser de 3000000; de ellos habr 30000 de longitud 240, otros tantos de longitud 241, etc. Luego
S ( A) = 30000 (240 + 241 + 242+....+339) = 30000

100 ( 240 + 339) = 868500000 unidades. 2

Este es el valor mnimo buscado.

2.- Recortamos varios crculos de papel (no necesariamente iguales) y los extendemos sobre una mesa de modo que haya algunos solapados (con parte interior comn), pero de tal forma que no haya ningn crculo dentro de otro. Prueba que es imposible ensamblar las piezas que resultan de recortar las partes no solapadas y componer con ellas crculos distintos. Solucin: La frontera de las piezas recortadas (que no sean crculos completos) est formada por arcos cncavos y convexos (vistos desde fuera) que se cortan en puntos que llamaremos vrtices. En un vrtice pueden concurrir dos arcos cncavos o uno cncavo y otro convexo, pero nunca dos convexos ya que stos nicamente provienen de la frontera de los crculos iniciales. Adems, los ngulos que forman los arcos en cada vrtice no son de 0 ni de 180 ya que excluimos las tangencias interiores. Supongamos que tenemos un crculo obtenido ensamblando piezas recortadas. Existe al menos un punto P de la frontera de dicho crculo en el que concurren tres o ms arcos de la frontera de las pieza ensambladas (P es vrtice de dos o ms piezas). La tangente al crculo en P deja a todos los arcos en un mismo semiplano. Elegido un sentido de rotacin en P a partir de la P tangente, y avanzando en este sentido, el primer arco que encontramos es convexo y el ltimo cncavo. Por lo tanto es necesario que existan dos arcos consecutivos uno convexo y el otro cncavo los cuales forman parte de la frontera de una de las piezas ensambladas. Como el arco que forman dichas piezas no puede ser ni 0 ni 180, el punto P es un vrtice de la pieza. Esto es contradictorio pues en ningn vrtice pueden concurrir dos arcos convexos vistos desde fuera. Nota. Hay que entender en el enunciado.que quedan excluidas las tangencias interiores. De no ser as pueden encontrarse contraejemplos como el despiece que se muestra en la figura:

3.- Por el baricentro G de un tringulo ABC se traza una recta que corta al lado AB en P y al lado AC en Q. Demuestra que: PB QC 1 PA QA 4 Solucin: Dupliquemos el tringulo trazando AD paralela a BC y CD paralela a BA como muestra la figura y tomemos la longitud del lado AB como unidad. Llamando M a la interseccin de CD con la recta PQ y x = PB; 1-x = AP, tenemos:
D A

Por semejanza de AQP y QMC:

QC MC MC = = QA AP 1 x
PB GB 1 = = . MD GD 2

Q G P C B

Por semejanza de GPB y GMD:

Luego: MD = 2x y MC = 1 - 2x. Sustituyendo en el primer miembro de la relacin del enunciado queda:

PB QC 1 x (1 2 x ) 1 2 9 x 2 6 x + 1 0 ( 3x 1) 0 2 PA QA 4 4 (1 x )
Relacin vlida para cualquier x. La igualdad se alcanza para PB = x =
1 1 MC = PQ paralela al lado BC. 3 3

4.- Halla las soluciones enteras de la ecuacin: p(x + y) = xy siendo p un nmero primo. Solucin: Ya que p es primo, p 0 y p 1. De la ecuacin resulta que p divide a x o p divide a y. Como la ecuacin es simtrica respecto de x e y, si (, ) es solucin, tambin lo ser (, ). Si p divide a x, x = pa, (aZ) la ecuacin se puede poner como: pa p( pa + y)) = pay pa + y = ay y = ya que a es entero a 1 adems a y a - 1 son primos entre s, luego a - 1 divide a p. Al ser p primo slo hay cuatro posibilidades: a - 1 = 1 y a - 1 = p. Examinemos todos los casos. i) a - 1 = -1, entonces a = 0, x = 0, y = 0 2p ii) a - 1 = 1, entonces a = 2, x = 2p y = = 2p . 21 p( p + 1) iii) a - 1 = p, entonces a = p + 1, x = p(p + 1) y = = p+1. p + 1 1 p( 1 p ) = p1. iiii) a - 1 = -p, entonces a = 1 - p, x = p(1 - p) y = 1 p 1 En resumen las soluciones son: (0, 0); (2p, 2p); (p(p+1), p+1); (p(1-p), p-1); y por la simetra aadimos (p+1, p(p+1)); (p-1, p(1-p)).

RESPUESTAS DE LA XXXII OLIMPIADA ESPAOLA

os naturales a y b son tales que: de a y b no es mayor que


Solucin: Se tiene:

a+1 b+1 es entero. Demostrar que el mximo comn divisor + b a a+b

a + 1 b + 1 a2 + b2 + a + b + = . b a ab Sea d = m.c.d (a,b). Como ab es divisible por d2, entonces a 2 + b 2 + ab es divisible por d2 y tambin lo son a2 + b2 y a + b, y al ser a y b naturales, se tiene :

a + b d2 a + b d 2.- Sea G el baricentro del tringulo ABC. Si se verifica: AB + GC = AC + GB demostrar que el tringulo es issceles.
Solucin: Primera solucin. Teniendo en cuenta el teorema de la mediana, la relacin del enunciado se escribe:

2 a 2 + c2 b2 cb= 3 2 4

a 2 + b2 c2 2 4

multiplicando y dividiendo por la expresin conjugada queda: 3 2 c b2 ) 2 4( c + b cb= (c b) m c + m b =0 2 3 mc + m b Probaremos que el segundo factor es positivo, de donde se deduce la conclusin. Llamando B y C a los puntos medios de AC y Ab respectivamente, en los tringulos CCA y BBA tenemos por la desigualdad triangular: b c m b + > c; m c + > b . 2 2 Sumando ambas desigualdades se obtiene el resultado. Segunda solucin.

Llamando A, B, C a los puntos medios de los lados BC, AC y AB respectivamente y dividiendo por dos la condicin del enunciado podemos escribirla como: C' A C' G B' A B' G B' M C' + = + , 2 2 2 2 es decir los puntos C y B estn en una elipse de focos G A y G. C A' B Llamando M al punto medio de CB , M esta en la mediana AA y no es el centro de la elipse (punto medio del segmento AG), por tanto CB ha de ser perpendicular a AA, y entonces AA adems de mediana es altura y el tringulo es issceles.
A

3.- Sean a, b, c nmeros reales. Se consideran las funciones: f (x) = ax 2 + bx + c, Sabiendo que
f (1) 1, f (0) 1, f (1) 1,

g(x) = cx 2 + bx + a .

demostrar que si -1 x 1, entonces: f (x) 5 4 y g(x) 2.

Solucin: Podemos conseguir coeficientes A, B, c tales que se tenga idnticamente: f (x) = Ax(x + 1) + Bx(x 1) + C(x 2 1) . Particularizando para x = 1, -1 ,0 y resolviendo el sistema queda: f (1) f ( 1) f ( x) = x( x + 1) + x( x 1) + f (0)(1 x 2 ) x R 2 2 De aqu se deduce: 1 1 f (x) x(x + 1) + x(x 1) + 1 x 2 ; 2 2 como 1 x 1, 1 + x 0, 1 x 0 y 1 x 2 0 , resulta x x 5 1 5 f (x) (1 + x) + ( 1 x) + 1 x 2 = x 2 + x + 1 = x , 2 2 4 2 4 1 por otra parte, para x 0, g( x) = x 2 f . Entonces x g (x ) = vlido para 1 x 1 . As pues g (x ) 1+ x 1 x + + 1 x2 = 2 x2 2 2 2 f (1) f (1) (1 + x) + (1 x) + f (0)( x 2 1) 2 2
2

4.- Discutir la existencia de soluciones de la ecuacin


x2 p + 2 x2 1 = x segn los valores del parmetro real p, y resolverla siempre que sea posible.

Solucin:

Si p < 0, entonces

x 2 p > x ; como 2 x 2 1 > 0 , no existe solucin. Por tanto p 0.


8( 2 p) x 2 ( 4 p) , de donde x =
2

Aislando un radical y elevando al cuadrado dos veces se llega a la ecuacin:


4p 8( 2 p)

Como x R, p < 2, as que x= Sustituyendo en la ecuacin dada se obtiene 4p 8( 2 p) .

( 4 3p) 8( 2 p)
como p > 0, |p| = p; y finalmente:

+2

8( 2 p)

p2

4p 8( 2 p) 4 3

4 3p + 2 p = 4 p 4 3p = 4 3p 4 3p 0 0 p

5.- En Port Aventura hay 16 agentes secretos. Cada uno de ellos vigila a algunos de sus colegas. Se sabe que si el agente A vigila al agente B, entonces B no vigila a A. Adems, 10 agentes cualesquiera pueden ser numerados de forma que el primero vigila al segundo, ste vigila al tercero,....., el ltimo (dcimo) vigila al primero. Demostrar que tambin se pueden numerar de este modo 11 agentes cualesquiera.
Solucin: Diremos que los agentes A y B son neutrales si A no vigila a B ni B vigila a A. Sean A1, A2,........An los agentes. Sean: ai el nmero de agentes que vigilan a Ai. bi el nmero de agentes que son vigilados por Ai. ci el nmero de agentes que son neutrales con Ai. Es claro que a i + b i + c i = 15, a i + c i 8, b i + c i 8 i = 1,2, ....16

Notemos que si una cualquiera de las dos ltimas desigualdades no se verificase, entonces no se podran numerar 10 espas en la forma indicada. Combinando las relaciones anteriores obtenemos ci 1. Por tanto para cualquier espa el nmero de sus colegas neutrales es 0 1. Razonemos por reduccin al absurdo. Supongamos que hubiera un grupo de 11 espas que NO se pudiera numerar en la forma descrita. Sea B uno cualquiera de los espas de este grupo. Numeramos los otros 10 espas como C1, C2, ....C10 de modo que C1 vigila C2....., C10 vigila a C1. Supongamos que ninguno de los Ci sea neutral respecto de B. Entonces si C1 vigila a B, B no puede vigilar a C2, pues en tal caso C1, B, C2, ....C10 formara un grupo en las condiciones del problema, luego C2 vigila B , etc . De este modo llegamos a la contradiccin de que todos los espas del grupo

vigilan a B. Por tanto cada uno de los 11 espas debe tener uno y solo uno del grupo neutral con l, lo cual es imposible. 6.- La figura de la izquierda se compone de seis pentgonos regulares de lado 1m. Se dobla por las lneas de puntos hasta que coincidan las aristas no punteadas que confluyen en cada vrtice. Qu volumen de agua cabe en el recipiente formado?. Solucin: La figura formada por el agua es un tronco de pirmide pentagonal cuya base menor es el pentgono dado y cuya base mayor es otro pentgono regular que tiene por lado la diagonal del anterior paralela a la arista de la base como se muestra en la figura inferior derecha. Ms abajo, se ha dibujado en forma invertida para una mejor comprensin del dibujo. (Figura central). Establezcamos primero algunas relaciones conocidas para un pentgono regular de lado 1. (Figura de la izquierda). Llamemos d a la diagonal. Por semejanza de los tringulos ABE y PCD tenemos: 1 d 1+ 5 = d2 d 1 = 0 d = = (1) d 1 1 2
A E h 72 O P D C R B B C R-r C H r A H-h 1 A

1 36 B

es el llamado nmero ureo y representa la relacin entre la diagonal y el lado de un pentgono regular. En nuestro caso es la relacin de semejanza entre las bases del tronco de pirmide. d 1+ 5 1 1 1 Adems : cos 36 = = = y para el radio r : sen 36 = r= = ( 2) . 2 2 2 4 2r 2 sen 36 4 Llamando V al volumen de la pirmide grande , v al de la pequea, sabemos que V = 3v ; y para el volumen del tronco de cono Vt queda: 1 Vt = V v = 3 v v = v( 3 1) = ah( 3 1) ; siendo a el rea del pentgono de lado 1. Slo nos 3 queda calcular a, h, sustituir y operar: El rea a la calculamos sumado 5 tringulos issceles de lados iguales r, r formando 72 5 5 5 5 a = r 2 sen 72 = r 2 2 sen 36 cos 36 = r cos 36 = r . (hemos usado 2rsen36 = 1 de (2)). 2 2 2 4 Para calcular h, por la semejanza de los tringulos de la figura central, tenemos:

r ( H h) r 1 ( R r ) 2 1 r 2 ( 1) 2 H h Hh = = h= = = = 1 R r Rr Rr r ( 1) Como verifica la ecuacin (1): 2 = +1; tenemos para la expresin de h:


1 h= ( 1) 2 4 2 1 = 4 2 2 + 2 1

( 1) 2 4 2

( 1)

4 2

3 2 2 + 2

( 1)

4 2

( 1)

4 2

Sustituyendo las expresiones de a y h y poniendo 3 -1= (-1)(2 + + 1); queda: ( 3 1) 15 5 ( 2 + + 1) 5 ( + 1) 5 2 + 1 Vt = = = = 3 4 4 2 ( 1) 4 2 12 6 3 6 3 4 2 y sustituyendo el valor de de (1), queda finalmente: Vt = 5 2 + 5 15 + 7 5 = 2,554m 3 3 5 5 12

RESPUESTAS DE LA XXXIII OLIMPIADA ESPAOLA


1.- Calcular la suma de los cuadrados de los cien primeros trminos de una progresin aritmtica, sabiendo que la suma de ellos vale -1, y que la suma de los trminos de lugar par vale +1. 2.- Un cuadrado de lado 5 se divide en 25 cuadrados unidad por rectas paralelas a los lados. Sea A el conjunto de los 16 puntos interiores, que son vrtices de los cuadrados unidad, pero que no estn en los lados del cuadrado inicial. Cul es el mayor nmero de puntos de A que es posible elegir de manera que TRES cualesquiera de ellos NO sean vrtices de un tringulo rectngulo issceles?. 3.- Se consideran las parbolas y = x2 + px + q que cortan a los ejes de coordenadas en tres puntos distintos por los que se traza una circunferencia. Demostrar que todas las circunferencias trazadas al variar p y q en R pasan por un punto fijo que se determinar.

Segunda Sesin

4.- Sea p un nmero primo. Determinar todos los enteros kZ tales que

k 2 kp es natural.

5.- Demostrar que en un cuadriltero convexo de rea unidad, la suma de las longitudes de todos los lados y diagonales no es menor que 2 2 + 2 .

6.- Un coche tiene que dar una vuelta a un circuito circular. En el circuito hay n depsitos con cierta cantidad de gasolina. Entre todos los depsitos contienen la cantidad exacta que el coche necesita para dar una vuelta. El coche comienza con el depsito vaco. Demostrar que con independencia del nmero, posicin y cantidad de combustible de cada depsito, siempre se puede elegir un punto de comienzo que le permita completar la vuelta. Notas: a) El consumo es uniforme y proporcional a la distancia recorrida. b) El tamao del depsito es suficiente para albergar toda la gasolina necesaria para dar una vuelta.

Soluciones

1.- Calcular la suma de los cuadrados de los cien primeros trminos de una progresin aritmtica, sabiendo que la suma de ellos vale -1, y que la suma de los trminos de lugar par vale +1.
Sea la progresin a, a + d, a + 2d, ......, a + 99d, entonces tenemos que hallar: S = a2 + (a + d)2 + (a + 2d)2 +.....+ (a + 99d)2 =100a2 + 2ad (1 + 2 + ...+ 99) +d2 (12 + 22 +....+ 992).
( a + a + 99d )50 = 1 que operado y resuelto sale: Para calcular a y d resolvemos el sistema: ( a + d + a + 99d ) 25 = 1 a = -2,98; d = 0,06. El resto es fcil de calcular. Los parntesis son progresiones de primer y segundo orden.

1 + 2 + ...+ 99 = 4950; 12 + 22 +....+ 992 = 328350. El resultado final es S = 299,98

2.- Un cuadrado de lado 5 se divide en 25 cuadrados unidad por rectas paralelas a los lados. Sea A el conjunto de los 16 puntos interiores, que son vrtices de los cuadrados unidad, pero que no estn en los lados del cuadrado inicial. Cul es el mayor nmero de puntos de A que es posible elegir de manera que TRES cualesquiera de ellos NO sean vrtices de un tringulo rectngulo issceles?.
Numeremos los puntos como indica la figura 13 9 5 1 14 10 6 2 15 11 7 3 16 12 8 4

Por simple tanteo se obtiene un conjunto de seis puntos verificando la condicin del enunciado, por ejemplo {1, 2, 3, 8, 12, 16}. Supongamos que hubiera un conjunto M de 7 puntos verificando la condicin del enunciado. Notemos que si cuatro puntos forman un cuadrado, a lo sumo figurarn dos de ellos en M. Los puntos de los conjuntos {1, 4, 16, 13}, {2, 8, 15, 9}, {3, 12, 14, 5} forman cuadrados y su unin forma el contorno exterior de A, luego a los sumo 6 de los puntos elegidos deben estar en M y por tanto al menos un punto de M debe ser del conjunto interior de A: {6, 7, 10, 11}. Por la simetra de la figura supongamos que es el 7. Como {7, 16, 9} y {1, 7, 14} forman tringulos rectngulo issceles, a lo sumo 2 de los puntos del conjunto {1, 9, 14, 16} debern figurar en M. Adems {5, 7, 13, 15} forman un cuadrado por tanto a lo sumo podremos elegir dos nmeros entre {5, 13, 15} , de ello se deduce en M deben figurar al menos tres puntos de {2, 3, 4, 6, 8, 10, 11, 12} . Si descomponemos este conjunto en dos subconjuntos cuadrados y disjuntos : {3, 6, 11, 8} y {2, 4, 10, 12} forzosamente de uno de ellos habremos de tomar dos puntos y uno de otro.

Si tomamos dos puntos del primero las nicas posibilidades son {3, 11} y {6, 8} ambas incompatibles con cualquier eleccin del punto restante en el segundo conjunto. Si los dos puntos se eligen del segundo las nicas maneras son {2, 12} y {4, 10}, de nuevo incompatibles con cualquier eleccin del punto que falta en el primer conjunto. En resumen el nmero mximo de elementos es 6.

3.- Se consideran las parbolas y = x2 + px + q que cortan a los ejes de coordenadas en tres puntos distintos por los que se traza una circunferencia. Demostrar que todas las circunferencias trazadas al variar p y q en R pasan por un punto fijo que se determinar.
1 Solucin (analtica) Sean y las races . Los tres puntos que definen la circunferencia son A(, 0); B(, 0); C(0, q). Verificando + = - p. y = q. (1) p La mediatriz de AB es la recta paralela al eje OY de ecuacin x = . 2 Hallando la mediatriz de AC, cortando con la anterior y teniendo en cuenta (1) se obtiene para el p q + 1 centro las coordenadas , y para el radio r = 2 2
2

p 2 + (1 q ) . La ecuacin de la 4
2

2 2 p 2 + (1 q ) p q + 1 circunferencia es: x + + y , que una vez operada queda: = 2 2 4

x 2 + y 2 + px (1 + q ) y + q = 0

que se verifica para el punto (0, 1) con independencia de p y q como se comprueba por simple sustitucin. Claramente el punto fijo se puede obtener a partir de tres circunferencias concretas. 2 Solucin (geomtrica). Puesto que la parbola corta al eje de abscisas en dos puntos, se podr escribir en la forma: y = (x - a) (x - b) y los puntos de interseccin son A(a, 0); B(b, 0); C(0,ab) La inversin de polo el origen que transforma A en B, transforma C en U(0, 1) , as que los cuatro puntos A,B,C,U son concclicos y todas las circunferencias pasan por el punto fijo U.

4.- Sea p un nmero primo. Determinar todos los enteros kZ tales que

k 2 kp es natural.

p p 2 + 4n 2 Solucin: Pongamos k kp = n k pk n = 0 k = 2 El radicando ha de ser cuadrado perfecto, llamsmole a. Se tiene:


2 2 2

(1) .

p2 + 4n2 = a2 p2 = (a+2n)(a-2n). Como p es primo y a + 2n a - 2n, slo hay dos posibilidades: 1) a + 2n = p2 y a - 2n =1 2) a + 2n = p y a - 2n = p

p2 + 1 p2 1 ; n= , lo que exige p 2 (n natural). 2 4 En el caso 2) resulta a = p ; n = 0. Sustituyendo los valores de a en (1) y operando queda: Si p = 2 , entonces k = 2 o k = 0 Su p 2 entonces quedan los cuatro valores: 2 2 p 1 p + 1 k1 = , k 3 = p, k 4 = 0 , k 2 = 2 2 En el caso 1) a =

5.- Demostrar que en un cuadriltero convexo de rea unidad, la suma de las longitudes de todos los lados y diagonales no es menor que 2 2 + 2 .

Solucin. 1
a d q b c p

Sea el cuadriltero de lados a, b, c, d y diagonales p y q. Trazando la paralelas por cada vrtice a la diagonal que no pasa por l se forma un paralelogramo de rea 2 y lado p y q. Por el teorema isoperimtrico, de todos los paralelogramos de rea 2, el cuadrado tiene permetro mnimo que vale 4 2 , luego
2( p + q ) 4 2 p + q 2 2

(1)

En cuanto al los lados por el mismo teorema para una cuadrado de rea 1 el permetro es 4 luego: a + b + c + d 4 (2)

Sumando(1) y (2) se obtiene el resultado. Solucin 2 (Sin usar la propiedad isoperimtrica). Consiste en establecer directamente las desigualdades (1) y (2). Si es el ngulo que forman las diagonales, tenemos: pq pq 1= sen pq 2 2 2 pero (p + q)2 = (p - q)2 + 4pq 4pq 8. de donde p + q 8 = 2 2 (1). Para los lados, si descomponemos el cuadriltero en dos tringulos mediante la diagonal q, tenemos: ab cd 1 + 2 2 Descomponiendo ahora en dos tringulos mediante la diagonal p resulta: 1 bc da + 2 2

y de ambas desigualdades se obtiene: ab + bc + cd + da 4. Pero:(a + b + c + d)2 = ((a + c) - (b + d))2 + 4 (a + c)(b + d) 4 (a + c)(b + d) 16, de donde a + b + c + d 4 (2) Basta sumar (1) y (2) para obtener lo pedido.

6.- Un coche tiene que dar una vuelta a un circuito circular. En el circuito hay n depsitos con cierta cantidad de gasolina. Entre todos los depsitos contienen la cantidad exacta que el coche necesita para dar una vuelta. El coche comienza con el depsito vaco. Demostrar que con independencia del nmero, posicin y cantidad de combustible de cada depsito, siempre se puede elegir un punto de comienzo que le permita completar la vuelta. Notas: a) El consumo es uniforme y proporcional a la distancia recorrida. b) El tamao del depsito es suficiente para albergar toda la gasolina necesaria para dar una vuelta.
Solucin 1 (Sergi Elizalde . Concursante)

Sean c1, c2, .....,cn las cantidades de combustible en cada uno de los n depsitos y sean d1, d2,....,dn las distancias a recorrer desde cada depsito hasta el siguiente. Hagamos el grfico del consumo comenzando en un punto de aprovisionamiento cualquiera. Notemos que los tramos inclinado tienen todos la misma pendiente. Los tramos bajo el eje

1 6 1 2 3 4 5 7 2

representan las situaciones imposibles. La pendiente de los tramos inclinados vale :

c d

i i

. La

hiptesis de que el total de combustible es la cantidad exacta para dar la vuelta se traduce en que la grfica comienza y termina en el eje OX. La funcin resultante (trazo continuo) tiene un mnimo, en la figura el punto 3. Basta comenzar en ese punto para asegurar que el recorrido es posible. En efecto, grficamente equivale a trasladar el eje OX en sentido vertical hasta el punto ms bajo con lo que aseguramos que ninguna zona queda bajo el eje. La nueva grfica puede trazarse a partir del punto 3 siguiendo el mismo trazado hacia la derecha y trasladando la parte anterior (tramos 1-2 y 2-3) al punto final de la grfica anterior (de puntos en la figura).
Solucin 2 (M A. Lpez Chamorro. Miembro del Jurado).

Se numeran los depsitos de 1 a n comenzando por uno cualquiera en sentido antihorario. Llamamos: a1, a2, ...,an a la cantidad de gasolina de cada depsito. b1, b2, ...,bn a la cantidad de gasolina necesaria para ir del depsito ai al siguiente. d1 = a1 -b1 , d2 = a2 - b2 ,....., dn = an - bn Diremos que un depsito es positivo o negativo segn lo sea di . Si di = 0 , la ubicacin del depsito i no influye en la ordenacin del recorrido. Por ello podemos suponer sin prdida de generalidad que di 0 para todo i. Por otra parte, si hay varios depsitos consecutivos positivos o negativos, el tramo limitado por ellos se puede considerar como un nico tramo positivo o negativo. As, el problema se reduce a tener un nmero par de depsitos alternativamente positivos o negativos. Agrupando los tramos por parejas, stas resultarn positivas o negativas y volvemos a repetir el proceso.

As reducimos el caso a un nmero de depsitos n1 < n/2. Como n < 2k , a lo sumo en k - 1 etapas llegaremos a tener 2 depsitos, uno con ms gasolina que otro, en cuyo caso empezando por el que tenga ms combustible se puede completar el circuito. El caso de un slo depsito es trivial. Se empieza y termina en ese nico depsito.

RESPUESTAS DE LA XXXIV OLIMPIADA ESPAOLA


1.- Un cuadrado ABCD de centro O y lado 1, gira un ngulo en torno a O. Hallar el rea comn a ambos cuadrados.
Solucin 1. Por la simetra bastar considerar 0 < < 90, ya que la funcin es peridica con periodo de un cuarto de vuelta. El rea pedida S() sale restando del rea del cuadrado cuatro tringulos como el PAM. Llamando x al cateto PA e y al cateto AM, el rea de cuatro tringulos vale 2xy. Como el lado BA vale 1, tenemos:

A' B B'
O

P A

D' C C' D

x + y = 1 x + y
2

(1)

relacin que elevada al cuadrado y simplificada queda: 2 xy = 1 2 x 2 + y 2

( 2)

pero x = x 2 + y 2 cos , y = x 2 + y 2 sen , y sustituyendo en (1) resulta:

x 2 + y 2 (1 + cos + sen ) = 1 x 2 + y 2 = sustituyendo en (2) y operando obtenemos: 2 xy = 1

1 1 + sen + cos

2 sen + cos 1 = . 1 + sen + cos sen + cos + 1

Finalmente para el rea pedida obtenemos: S( ) = 1 sen + cos 1 2 = con 0 90 sen + cos + 1 sen + cos + 1
A' B B'
O

Solucin 2. El rea pedida consta de 8 tringulos como el sombreado en la figura OPM. Tomando como base b = MP, la altura es constante (de trazos en la figura) y vale . En el tringulo PAM se tiene: MA = b cos , PA = b sen ; pero BM = MA y PA = PA, adems:

P A

D' C C' D

BM + MP + PA = 1 b cos + b + b sen = 1, de donde b= y el rea pedida es: 1 sen + cos + 1

S( ) = 8

11 1 2 = con 0 90 2 2 sen + cos + 1 sen + cos + 1

2.- Hallar todos los nmeros naturales de 4 cifras, escritos en base 10, que sean iguales al cubo de la suma de sus cifras. Solucin: Sea n un nmero verificando el enunciado, y s la suma de sus cifras. Como 1000 n 9999 y n = s3 , resulta

11 s 21 Si n = xyzt, tenemos:

(1)

1000x + 100y + 10z + t = s3 x+y+z+t=s restando queda: 999x + 99y + 9z = s3 - s

(2)

(3)

cuyo segundo miembro ha de ser mltiplo de 9 (por serlo el primero) y, habida cuenta de que s3 - s = (s - 1) s (s + 1) y por (1), slo hay tres valores de s3 - s que son mltiplos de 9: 161718; 171819 y 181920 sustituimos en (3) y analizamos cada caso. 1 999x + 99y + 9z = 161718 111x + 11y + z = 544 resulta inmediatamente x = 4; y = 9; z = 1, valores que llevados a (2) con s = 17 se obtiene t = 3 y finalmente n = 4913 2 999x + 99y + 9z = 171819 111x + 11y + z = 646 de donde x = 5; y = 8; z = 3, valores que llevados a (2) con s = 18 se obtiene t = 2 y finalmente n = 5832 3 999x + 99y + 9z = 181920 111x + 11y + z = 760 resulta x = 6; y = 8; z = 6, valores que llevados a (2) con s = 19 resulta una contradiccin. Resumiendo, las nicas soluciones son 4913 y 5832

3.- Se considera el tringulo ABC y su circunferencia circunscrita. Si D y E son puntos sobre el lado BC tales que AD y AE son, respectivamente, paralelas a las tangentes en C y en B a la circunferencia circunscrita, demostrar que:
BE AB = CD AC 2
Solucin: Los tringulos ABC y ADC son semejantes pues tienen los tres ngulos iguales ya que: ADC = BCM = BAC (la primera igualdad por ser AC y CM paralelas y la segunda por ser BCM ngulo semiinscrito) y el ngulo ACD es comn. Estableciendo la proporcionalidad entre sus lados, resulta:
A
2

CD AC 2 = CD BC = AC AC BC

(1)

De modo anlogo los tringulos ABC y ABE son semejantes pues: AEB = EBM = BAC y el ngulo ABE es comn. Estableciendo la proporcionalidad entre sus lados, resulta: BE AB 2 = BE BC = AB AB BC

( 2)

Dividiendo las igualdades (1) y (2) se obtiene el resultado.

4.- Hallar las tangentes de los ngulos de un tringulo sabiendo que son nmeros enteros positivos.
Solucin.

Sean , , los tres ngulos y supongamos . Si fuera entonces tg no es entero. Si tg > 1, entonces arc tg 2 > arc tg Por tanto tg = 1 y + = 3 , con lo que: 4 3=

, tendra que ser < y 2 4

, imposible porque + + = . 3

tg( + ) = 1 = relacin que operada se convierte en:

tg + tg 1 tg tg

(tg -1)(tg -1) =2 de donde, por ser enteros positivos, se sigue tg = 2 y tg = 3. Existe una visualizacin sin palabras de la solucin: arc tg 1 + arc tg 2 + arc tg 3 = .

5.- Hallar todas las funciones f : N N estrictamente crecientes y tales que:

f(n + f(n)) = 2 f(n) para n = 1, 2, 3, ...

Solucin: Supongamos f(1) = b. Entonces, f(1 + b) = 2b, como f es estrictamente creciente, se tiene:
b = f(1) < f(1 +1 ) < .< f(1+b) = 2b = b + b. y resulta que f(1), f(2),.f(1+ b) son b + 1 naturales, distintos, el primero vale b y el ltimo 2b, por tanto han de ser consecutivos. resulta entonces: f(1) = b, f(2) = 1 + b, f(3) = 2 + b,, f(1 + b) = b + b. En general, para n > 1, si f(n) = c , f(n + c) = 2c = c + c y resulta que: c = f(n) < f(n + 1) <.< f(n + c) = c + c y los nmeros f(n), f(n + 1), .., f(n + c) son consecutivos. As pues, f(n) = n - 1 + f(1)

6.- Determina los valores de n para los que es posible construir un cuadrado de n n ensamblando piezas del tipo:

Solucin: Evidentemente n2 debe ser mltiplo de 4 y, por tanto n necesariamente es par. Si n = 4k podemos dividir cualquier cuadrado n n en k2 sub-cuadrados del tipo 4 4 cada uno de los cuales lo podemos rellenar en la forma sealada en la figura de la izquierda. Queda slo considerar el caso n = 4k + 2. Veamos que en ese caso la repuesta es negativa. Supongamos que fuera posible. Si pintamos cada cuadradito alternativamente de blanco y negro como en un tablero de ajedrez, hay dos posibilidades para cada pieza:

B B N B N

N B N

Sea a el nmero de piezas del tipo de las de la izquierda y b el nmero de piezas del tipo de las de la derecha. Tenemos: ( 4 k + 2) 2 2 a+b= = ( 2 k + 1) = 4 k 2 + 4 k + 1 4 luego a + b ha de ser impar. Por otra parte, como hay tantas casillas blancas como negras, se tiene: 3a + b = 3b + a a = b, de donde a + b = 2a ha de ser par en contradiccin con lo anterior.

5.- Demuestra que en el caso de que las ecuaciones: nx3 - 2 m2 x2 - 5mnx - 2m3 - n2 = 0 (n 0) x3 + mx - n = 0 , tengan una raz comn, la primera tendr dos races iguales y determina entonces las races de las dos ecuaciones en funcin de n.

Solucin:
Sea la raz comn de ambas ecuaciones. Entonces 3 + m = n (1) y sustituyendo en la segunda ecuacin se obtiene, tras hacer operaciones: 6m 4 + 8m2 2 +2m3 = 0 Supongamos m 0, entonces simplificando la relacin anterior queda: (2) 3 4 + 4m 2 +m2 = 0

2 (i) Resolviendo (2) respecto de m obtenemos m = Analicemos cada caso. 2 -3 (ii) (i) Si m = -2, sustituyendo en la primera ecuacin y despejando n queda: n = 3 - 3 = 0 en contra de lo supuesto. Por tanto (i) queda descartado. (ii) Si m = -33, sustituyendo en la primera ecuacin y despejando n queda: n = 3 -3 3 = -2 3 y la primera ecuacin queda: 2 x 3 3 2 x + 2 3 = ( x ) ( x 2 + x 2 2 ) = ( x ) ( x + 2) que, efectivamente, tiene la raz doble. n de n = -2 3 obtenemos = 3 . 2 Entonces la segunda ecuacin es de la forma 2 3 ( x 3 + 9x 2 + 15 2 x + 25 3 ) = 0 ,
y, dividiendo por (x - ) resulta 2 3 ( x )( x + 5) = 0 , cuyas races son y 5 siendo doble la ltima. Si m = 0, las dos ecuaciones son iguales y sus tres races son las mismas pero la primera no tiene dos races iguales por lo que en el enunciado debera haberse aadido m 0.
2

C'

C''

A A''

C B'' P

6.- En la figura, AB es un segmento fijo y C un punto variable dentro de l. Se construyen tringulos equilteros de lados AC y CB, ACB y CBA en el mismo semiplano definido por AB, y otro de lado AB, ABC en el semiplano opuesto. Demuestra: a) Las rectas AA, BB y CC son concurrentes. b) Si llamamos P al punto comn a las tres rectas del apartado a), hallar el lugar geomtrico de P cuando C vara en el segmento AB. c) Los centros A, B y C de los tres tringulos forman un tringulo equiltero. d) Los puntos A, B, C y P estn sobre una circunferencia.

A'

B'

Solucin:
a) Se traza la circunferencia circunscrita al tringulo ABC y se llama P a la interseccin de CC con ella. Evidentemente (arco capaz) APB=120 y PC es su bisectriz con lo que APC=CPB=60 y P ha de estar en las circunferencias circunscritas a los tringulos ACB y BCA. Por tanto las tres circunferencias se cortan en P. Como CPB=120 y CPB=60 sumando queda: BPB=180 y P est alineado con BB. De modo anlogo se ve que P est alineado con AA.
A C'

C''

C B'' A'' P A'

b) Como P est definido por la interseccin de la recta CC con la circunferencia circunscrita al tringulo ABC el lugar pedido es el arco APB de esa circunferencia. c) Los lados del tringulo son perpendiculares a las cuerdas PA , PB y PC que forman ngulos de 60 o 120. por ello, entre s forman ngulos iguales de 60.

B'

d) Basta comprobar que los centros C, B, A y P verifican el teorema de Tolomeo:


PC ' ' A' ' B ' ' = PA' ' B ' ' C ' ' + PB ' ' A' ' C ' ' PC ' ' = PA' ' + PB' ' AB = AC + CB

siendo la ltima igualdad evidente por construccin.

RESPUESTAS DE LA XXXV OLIMPIADA ESPAOLA


Problema 1.
Las rectas t y t, tangentes a la parbola de ecuacin y = x2 en los puntos A y B, se cortan en el punto C. La mediana del tringulo ABC correspondiente al vrtice C tiene longitud m. Determinar el rea del tringulo ABC en funcin de m.
Y t

t' M A

Solucin: Sean A(a, a2) ; B(b, b2) . Las ecuaciones de t y t' son:
t: y = 2ax - a2 , t': y = 2bx - b2

B X

a+b y su interseccin C es: C , ab . 2 a+b , paralela al eje La mediana CM est en la recta: x = 2 a + b a2 + b2 OY. Las coordenadas de M son: . 2 , 2
Tenemos: m = CM

ba = 2 2 Poniendo [XYZ ] para denotar el rea del tringulo de vrtices X,Y,Z queda finalmente:

2 ( a b) = y si h es la altura del tringulo

BMC resulta: h =

m 2

[ABC ] = 2[BMC ] = 2 1 m
2

m = 2

m3 2

Problema 2.
Probar que existe una sucesin de enteros positivos a1, a2,, an, tal que

a12 + a22 +.+ an2


es un cuadrado perfecto para todo entero positivo n.

2 En efecto, k 2 = p 2 a n +1 = ( p + a n +1 )( p a n 1 ) . Pongamos a = p + a n +1 ; b = p a n +1 . a+b a b a+b ab Tenemos: p = ; k2 = . ; a n +1 = 2 2 2 2 La ltima expresin exige que a y b son de la misma paridad. Distinguiremos dos casos 1.- a y b son pares, entonces k2 = 4m . Tomado a = 2m; b = 2 queda:

Solucin: Lo haremos por induccin sobre n, para n = 2 basta tomar a1 = 3; a2 = 4 con 32 + 42 = 52. Supongamos que a12 + a22 +.+ an2 = k2 . Veamos que podemos encontrar un entero positivo an+1 2 2 tal que k 2 + a n +1 = p .

p = m +1 =

k2 k2 + 1; a n +1 = m 1 = 1 4 4

2.- a y b son impares, entonces k2 = 2m + 1. Tomando a = 2m +1, b = 1 queda:

p = m +1 =

k 2 1 k 2 1 + 1; a n +1 = m = 2 2

En ambos casos hemos encontrado an+1 entero verificando el enunciado.

Segunda Sesin Problema 3.


Sobre un tablero en forma de tringulo equiltero como se indica en la figura; se juega un solitario. Sobre cada casilla se coloca una ficha. Cada ficha es blanca por un lado, y negra por el otro. Inicialmente, slo una ficha, que est situada en un vrtice, tiene la cara negra hacia arriba; el resto de las fichas tiene la cara blanca hacia arriba. En cada movimiento se retira slo una ficha negra del tablero y se da la vuelta a cada una de las fichas que ocupan una casilla vecina. Casillas vecinas son las que estn unidas por un segmento. Despus de varios movimientos ser posible quitar todas las fichas del tablero?

Solucin: En el tablero, hay casillas de tres tipos : vrtice, lado, o interiores. Cada una de ellas tiene, respectivamente, dos, cuatro o seis casillas vecinas. Si pudiramos retirar todas las fichas del tablero, habra un momento en que quedara sobre l una nica ficha negra. Esa ficha era inicialmente blanca, luego ha tenido que cambiar de color un nmero impar de veces. Pero esto es imposible, porque una ficha se vuelve cada vez que se retira una ficha vecina, y ninguna ficha tiene un nmero impar de casillas vecinas.

Problema 4.
Una caja contiene 900 tarjetas, numeradas del 100 al 999. Se sacan al azar (sin reposicin) tarjetas de la caja y se anota la suma de los dgitos de cada tarjeta extrada. Cul es la menor cantidad de tarjetas que se deben sacar, para garantizar que al menos tres de esas sumas sean iguales?

Solucin: Hay 27 posibles resultados para la suma de dgitos (de 1 a 27). Las sumas 1 y 27 slo se puede obtener de un modo (100 y 999) En el caso ms desfavorable al sacar 52 (27 + 25) tarjetas todas repetirn suma dos veces y en la siguiente (extraccin 53) una de ellas aparecer por tercera vez. Por tanto el nmero pedido es 27 + 25 + 1 = 53.

Problema 5.
El baricentro del tringulo ABC es G. Denotamos por g a , g b , g c las distancias desde G a los lados a, b y c respectivamente. Sea r el radio de la circunferencia inscrita. Probar que: 2r 2r 2r i) g a , g b , g c 3 3 3

ii)

ga + gb + gc 3 r

Solucin 1 (del autor del problema):


A

gc G

b gb

i) Es sabido que uniendo G con cada vrtice, se forman tres tringulos BGC de base a y altura ga , AGC de base b y altura gb y AGB de base c y altura gc de la misma rea. Por tanto, llamando S al rea de ABC: aga = bgb = cgc = 2S (1) 3

ga a

(basta unir el incentro con los tres vrtices y quedan tres tringulos de bases a, b, c y altura comn r). Sustituyendo 2S en (1), y despejando queda: r a+b+c r a+b+c r a+b+c ; gb = ; gc = (2) 3 3 3 b c a a+b+c b+c 2r y por la desigualdad triangular (b + c a) , resulta : = 1+ 2 , de donde g a y a a 3 de modo anlogo para gb y gc . ga = b) De (2) , haciendo los inversos y sumando resulta:
1 1 1 3a 3b 3c 3 + + = + + = g a g b g c r ( a + b + c) r ( a + b + c) r ( a + b + c) r

C Por otra parte sabemos que r(a + b + c) = 2S

finalmente, aplicando la desigualdad entre las medias aritmtica y armnica:


ga + g b + gc g + g b + gc 3 3 = =r a 3 1 1 1 3 r 3 + + ga gb gc r Nota.- Sumando las tres desigualdades de a) slo obtenemos ga + gb + gc 2 r

Solucin 2 (de Ramn Jos que mereci mencin especial)


i) Consideremos los puntos MA, HA, GA como indica la figura. Pondremos hA a la altura correspondiente a A, p el semipermetro y S el rea de ABC. Los tringulos AMAHA y GMAGA son semejantes A siendo la razn de semejanza 3 (propiedad del baricentro sobre cada mediana). Entonces
hA G gA B M A GA HA C

hA = 3 gA (1) Por la desigualdad triangular:

b + c a 2 p 2a p a

a 1 p

multiplicando por hA y teniendo en cuenta (1) queda: ah 2S gA A gA 3p 3p 2 finalmente, como S = pr resulta g A r . 3 Anlogamente obtendramos las correspondientes desigualdades para gB y gC . ii) Usaremos la desigualdad x + 1 2 2 que se deduce de la obvia (x 1) 0 . (Consideraremos x

siempre x positivo). Tenemos entonces: a b b c c a + + + + + 6 b a c b a c Sumando 3, ordenando y operando resulta: 1+


a a b b c c 1 1 1 1 1 1 1 1 1 + + + 1 + + + + 1 9 a + + + b + + + c + + 9 b c a c a b a b c a b c a b c

sacando factor comn, dividiendo por 3 y poniendo 2p = a + b + c, queda: 2p 2p 2p + + 3 (2) 3a 3b 3c Por otra parte, como 3g a = h A ; 3 g b = hB ; 3g c = hC , resulta 2S = 3 g a a = 3g b b = 3g c c Despejando 3a, 3b y 3c y sustituyendo en (2), queda:

(g a + g b + g c ) p 3
S

Finalmente usando de nuevo S = pr, resulta

ga + gb + gc 3 r

Problema 6.

Se divide el plano en un nmero finito de regiones N mediante tres familias de rectas paralelas. No hay tres rectas que pasen por un mismo punto. Cul es el mnimo nmero de rectas necesarias para que N>1999?
Solucin: Supongamos que hay x rectas en la primera familia, y en la segunda y z en la tercera. Las x rectas de la primera familia determinan x + 1 regiones. La primera recta de la segunda familia determina en el plano (x +1)2 regiones, la segunda (x +1)3..... la y -sima determina (x + 1)(y + 1) regiones. La primera recta de la tercera familia es cortada por las x + y rectas existentes en x + y + 1 partes y cada una de estas partes divide a en dos a cada regin existente de modo que el nmero de regiones se incrementa en x + y + 1 regiones. Cada recta de la tercera familia aumenta las regiones existentes en la misma cantidad; luego el nmero total de regiones N vale:
N = (x + 1)( y + 1) + z ( x + y + 1) = x + y + z + xy + xz + yz + 1 = n + m + 1

con n = x + y + z y m = xy + xz + yz . Tenemos: 1 2 2 2 m = x 2 + y 2 + z 2 ( y z ) + ( z x ) + ( x y ) x 2 + y 2 + z 2 , entonces 2 n2 n2 n 2 = x 2 + y 2 + z 2 2m 3m m y N = n + m +1 n + +1. 3 3 n2 + 1 > 2002 . As, si n = 76 = x + y + z con x = 26, y = 25, z = 25, resulta: Para n = 76, n 2 + 3 m = 1925 y N = 2002.

RESPUESTAS DE LA XXXVI OLIMPIADA ESPAOLA


Problema 1. Sean los polinomios:

P(x) = x4 + ax3 + bx2 + cx + 1; Q(x) = x4 + cx3 + bx2 + ax + 1. Halla las condiciones que deben cumplir los parmetros reales a, b y c (a c) para que P(x) y Q(x) tengan dos races comunes y resuelve en ese caso las ecuaciones P(x) = 0; Q(x) = 0.
Solucin de Virginia Garca Madurga de Zaragoza.

Las races comunes a ambos polinomios sern races de la diferencia: P(x) - Q(x) = (a -c) x3 + (c - a) x Resolvemos la ecuacin P(x) - Q(x) = 0, sacando primero x factor comn: x (a c )x 2 + (c a ) = 0 Las tres races son: 0, 1 y -1, entre ellas tienen que estar las races comunes Como 0 no es raz ni de P(x) ni de Q(x), las dos races comunes tiene que ser 1 y -1. Sustituyendo estos valores en P(x) y Q(x) obtenemos el sistema: 2 + a + b + c = 0 2 a + b c = 0 que nos da las condiciones: b = -2 a = -c Los polinomios quedan en la forma: P(x) = x4 + ax3 - 2x2 - ax + 1 Q(x) = x4 - ax3 - 2x2 + ax + 1 Para resolver las ecuaciones P(x) = 0, Q(x) = 0, separamos por Ruffini las races conocida 1 y -1 y quedan las ecuaciones en la forma: P(x) = (x + 1)(x - 1) (x2 + ax - 1) = 0 Q(x) =(x + 1)(x - 1) (x2 - ax - 1) = 0 Resolviendo las ecuaciones de segundo grado queda finalmente: Soluciones de P(x) = 0: x = 1; x = -1; x = Soluciones de Q(x) = 0: x = 1; x = -1; x =
a + a2 + 4 a a2 + 4 ; x= 2 2
a + a2 + 4 a a2 + 4 ; x= 2 2

Problema 2.
B

La figura muestra un plano con calles que delimitan 12 manzanas cuadradas. Una persona P va desde A hasta B y otra Q desde B hasta A. Ambas parten a la vez siguiendo caminos de longitud mnima con la misma velocidad constante. En cada punto con dos posibles direcciones a tomar, ambas tienen la misma probabilidad. Halla la probabilidad de que se crucen.

Solucin de Fernando Cruz Robledillo (Madrid 2).

Definamos un sistema de coordenadas con origen en A y unidad el lado de un cuadrado. Como P y Q recorren caminos de longitud mnima, P slo puede ir a la derecha o arriba y Q a la izquierda o abajo. Todos los caminos tienen longitud 7, P y Q slo se podrn encontrar entre el 3 y el 4 movimiento, se han marcado en rojo todas las posibles posiciones de P tras el B tercer movimiento y en verde las de Q. Caso 1. P llega a (0, 3). 1 1 1 1 La probabilidad de que P llegue a (0, 3) es: = 2 2 2 8 Slo se puede cruzar con Q si ste est en (1, 3) lo que sucede 1 1 1 1 tambin con probabilidad = 2 2 2 8 A P est obligado a pasar a (1, 3) pero Q pasa a (0, 3) con 1 probabilidad . 2 111 1 La probabilidad de que se crucen entre (0, 3) y (1, 3) es: = 8 8 2 27 Caso 2. P llega a (1, 2).
3 1 La probabilidad de que P llegue a (1, 2) es 3 = (hay tres modos de llegar (1, 2)). 8 2 Slo se puede cruzar con Q si ste est en (1, 3) o en (2, 2). Distingamos ambos casos: 1 a) Q llega a (1, 3) con probabilidad , entonces se cruzarn entre (1, 2) y (1, 3) si P se mueve hacia 8 1 (1, 3) y Q hacia (1, 2) ambos movimientos con probabilidad . 2 311 1 3 La probabilidad de cruzarse es = 8 8 2 2 28 3 b) Q llega a (2, 2) con probabilidad , entonces se cruzarn entre (1, 2) y (2, 2) si P se mueve hacia 8 1 (2, 2) y Q hacia (1, 2) ambos movimientos con probabilidad . 2 3311 9 La probabilidad de cruzarse es = 8 8 2 2 28 Caso 3. P llega a (2, 1).
3

Procediendo de modo anlogo, la probabilidad de cruzarse entre los puntos (2, 1) y (2, 2) es: la de cruzarse entre (2, 1) y ( 3, 1) es 9 . 28

9 y 28

Caso 4. P llega a (3, 0). La probabilidad de cruzarse entre (3, 0) y (3, 1), es 1 . 27 La probabilidad pedida es la suma de todos los caso, resulta: entre (3, 0) y (4, 0) es 1 3 9 9 9 3 1 37 + 8 + 8 + 8 + 8 + 8 + 7 = 7 256 2 2 2 2 2 2 2
Problema 3.

3 y la de cruzarse 28

Dos circunferencias secantes C1 y C2 de radios r1 y r2 se cortan en los puntos A y B. Por B se traza una recta variable que corta de nuevo a C1 y C2 en dos puntos que llamaremos Pr y Qr respectivamente. Demuestra la siguiente propiedad: Existe un punto M, que depende slo de C1 y C2, tal que la mediatriz del segmento PrQr pasa por M.
Solucin de Luis Emilio Garca Martnez (Valencia U. Politcnica):

Sea O el punto medio del segmento M1M2. demostrar que todas las mediatrices de los segmentos PrQr pasan por el simtrico de B respecto de O. Sean = PrBM1; = M1BM2, Entonces: M2BQr = 180 - ( + ) y como el tringulo M2BQr es issceles, BM2Qr = 180 - 2 M2BQr = -180 + 2 ( + ) y por tanto,
Pr

A
C2

M1

M2 O

C1

B Qr

MM2Qr = 180 - + BM2Qr = 180 - - 180 + 2 ( + ) = + 2 De modo anlogo, por ser el tringulo PrM1B issceles, se tiene: PrM1B = 180 - 2 y PrM1M = 360 - (PrM1B + 180 - ) = 360 - 180 +2 -180 + = 2 + Resulta que para cualquier posicin de la recta variable los tringulos MM1Pr y MM2Qr son iguales y por tanto MPr = MQr y M est en la mediatriz de PrQr. Como M no depende de la recta variable queda probada la propiedad del enunciado.
Problema 4.

Encuentra el mayor nmero entero N que cumpla las siguientes condiciones : N a) E tiene sus tres cifras iguales. 3

N b) E es suma de nmeros naturales consecutivos comenzando en 1, es decir, existe un natural n 3 N tal que E = 1 + 2 + ....+ (n-1) + n . 3 Nota: E(x) es la parte entera de x.
Solucin de Roberto Alonso Prez del Pas Vasco.

N Condicin a): z = E = 111k; k N; 1 k 9 3 n (n + 1) 1 + 1 + 8z N Condicin b): z = E = 1 + 2 + 3 + ... + n z = n 2 + n 2z = 0 n = 2 2 3 (la otra raz es negativa). Juntando las dos condiciones, queda: n= 1 + 1 + 8111k 2

Como n es natural, el radicando ha ser cuadrado perfecto lo que ocurre slo para k = 6 que sustituido en la expresin anterior resulta n = 36. Recuperando la condicin a): N N z = E = 1116 = 666 667 > > 666 2001 > N > 1998 3 3 Por tanto el mayor N que cumple a) y B es N = 2000
Problema 5.

Tomemos cuatro puntos situados en el interior o el borde de un cuadrado de lado 1. Demuestra que al menos dos de ellos estn a distancia menor o igual que 1.
Solucin de Manuel Prez Molina del Alicante.

Vamos a demostrarlo por reduccin al absurdo. Supongamos que distribuimos 4 puntos en el cuadrado de manera que cada una de las seis distancias se mayor que 1. Entonces hay dos posibilidades: a) Los cuatro puntos forman un cuadriltero convexo. b) Los cuatro puntos forman un cuadriltero no convexo. Veamos ambos casos: P1 a) sean , , , los ngulos del cuadriltero convexo. Sabemos que + + + = 360. Adems cualquier pareja de puntos del interior (o frontera) del cuadrado estn a una distancia d 2 ya que el dimetro de dicho cuadrado es 2 . De la condicin + + + = 360, se deduce que P2 necesariamente uno de los ngulos ha de ser mayor o igual que 90, digamos por ejemplo 90. Tenemos (ver figura): Pi Pj > 1 , i j luego
P3

P4

P1P3 = P1P2 + P2 P3 2P1P2 P2 P3 cos como el cuadriltero es convexo, 90 180 y por tanto cos 0 y en consecuencia: P1 P3 P1 P2 + P2 P3 > 2 P1 P3 > 2 lo que es imposible. b) Si se forma un cuadriltero no convexo podemos elegir tres de los cuatro puntos formando un tringulo de modo que el cuarto punto sea interior. Supongamos que el punto interior es P4. Cada lado de dicho tringulo es menor o igual que 2 (dimetro del cuadrado) y por tanto estar contenido en un tringulo equiltero de lado 32 2 2 , y circunradio 2 = < 1 . Si su centro es C, P4 estar en el 2 2 3 3 interior de uno de los tres tringulos que resultan de unir C con cada C vrtice y la distancia de P4 a uno de los vrtices ser menor o igual que el 2 y por tanto menor que 1. circunradio, es decir menor que 3 hemos encontrado un par de puntos a distancia menor o igual que 1. Por ltimo si tres puntos estn alineados se reduce al caso b) y si los cuatro puntos estn alineados llamando x1, x2, x3 a las distancias entre puntos consecutivos, tenemos:
x1 + x 2 + x 3 2
2 2 2

y por el principio del palomar, uno de ellos, digamos x1, cumple: x 1


Problema 6.

2 < 1. 3

Demuestra que no existe ninguna funcin f : N N que cumpla: f(f(n)) = n + 1.


Solucin de Alberto Surez Real de Oviedo.

Supongamos que exista f : N N | f (f (n )) = n + 1 . Se tiene que f(0) = a N. Por el enunciado:


f (f (0 )) = 1; f (f (0 )) = f (a ) = 1

del mismo modo, f(1) = a + 1, f(a + 1) = 2, f(2) = a + 2,........ Supongamos que f(n - 1) = a + n - 1, entonces f( a + n -1) = a + n luego hemos probado por induccin que
f ((n )) = f (a + n ) = 2a + n

entonces, 2a + n = n + 1 a = 1 N 2

hemos llegado a una contradiccin y la condicin supuesta es falsa con lo que queda demostrado la inexistencia de la funcin f.

RESPUESTAS DE LA XXXVII OLIMPIADA ESPAOLA


Primera sesin

1.- Prueba que la grfica del polinomio P es simtrica respecto del punto A(a, b) s y slo s existe un polinomio Q tal que: P ( x ) = b + ( x a )Q (( x a ) 2 ), para todo x R.
Solucin: Supongamos primero que exista el polinomio P que cumple las condiciones requeridas. Sea x a = h x = a + h. Entonces : P( a h ) = b hQ ( h 2 ) P(a h ) + P(a + h ) y = b, para todo h R. Lo que significa 2 2 P( a + h ) = b + hQ ( h ) que la grfica de P es simtrica respecto del punto A( a, b). P(a h ) + P(a + h ) Sea x = a + h, P( x ) = P( a + h ) = R( h ). La condicin = b es equivalente a 2 R ( h ) + R( h ) = 2h, porque P (a h ) = R( h ). Para R ( h ) = a 0 + a1 h + ... + a n h n , la condicin anterior se escribe de la forma: a 0 + a1 h + ... + a n h n + a 0 a1 + a 2 h 2 ... + ( 1) n a n h n = 2b es decir a 0 + a 2 h 2 + ... + a m h m = b, para cada h R. m = n n par, m = n 1 n impar. Se deduce que a 2 = a 4 = .... = a m = 0, a 0 = b. Por tanto ahora se tiene que R ( h ) = b + a1 h + a 3 h 3 + ... y as existe un polinomio Q tal que
R ( h ) = b + hQ ( h 2 ), para algn polinomio Q. Por ltimo P ( x ) = R( h ) = b + ( x a )Q (( x a ) 2 ).
2.- Sea P un punto, en el interior del tringulo ABC, de modo que el tringulo ABP es issceles. Sobre cada uno de los otros dos lados de ABC se construyen exteriormente tringulos BCQ y CAR, ambos semejantes al tringulo ABP. Probar que los puntos P, Q, C y R o estn alineados o son los B vrtices de un paralelogramo.

A c R P a b

Solucin: Los tringulos ABC y PBQ son semejantes pues tienen un ngulo C Q igual ABC = PBQ y los lados que lo forman proporcionales: c BP = a BQ De modo anlogo, ABC es semejante a APR , por tanto PBQ y APR son semejantes (y al ser PB = PA son iguales). En particular: ARP = ACB y BQP = ACB Llamando = BAP = ABP, resulta:

QPR = 360 (180 2) (A + B) = 180 + 2 - (180 ACB ) = 2 + ACB QCR = ACB + 2 PRC = 180 2 ARP = 180 - 2 ACB PQC =180 2 BQP = 180 - 2 ACB
Las cuatro igualdades establecen que los dos pares de ngulos opuestos del cuadriltero PQCR son iguales y es un paralelogramo.

La alineacin es un caso particular y se producir cuando ACB + 2 = 180, es decir cuando 180 ACB = . 2 3.- Estn dados 5 segmentos de longitudes a1, a2, a3, a4 y a5 tales que con tres cualesquiera de ellos es posible construir un tringulo. Demuestra que al menos uno de esos tringulos tiene todos sus ngulos agudos. Solucin: Supongamos que 0 < a1 a 2 a3 a 4 a5 . Si ningn tringulo es acutngulo, tendramos:
2 2 a3 a12 + a 2 (1) 2 2 a2 2 + a 3 a 4 (2) 2 2 2 + a4 a5 a3 (3)

Pero (desigualdad triangular):


2 2 < a12 + a 2 + 2a1 a 2 (4) a5<a1 + a2, luego a5

Sumando las desigualdades (1),(2),(3) y (4) tenemos:


2 2 2 2 2 2 2 2 a12 + 2a 2 + 2a 3 + a4 + a5 < a3 + a4 + a5 + a12 + a 2 + 2a1 a 2

es decir,
2 2 a2 + a3 < 2a1 a 2
2 2 2 Como a 2 a3 , resulta 2a 2 a2 + a3 < 2a1 a 2 , y por tanto a2 < a1, en contradiccin con la ordenacin inicial.

Segunda sesin 4.- Los nmeros enteros desde 1 hasta 9 se distribuyen en las casillas de una tabla 3x3. Despus se suman seis nmeros de tres cifras: los tres que se leen en filas y los tres que se leen en columnas. Hay alguna distribucin para la cual el valor de esa suma sea 2001?

Solucin: Consideremos la distribucin:


a b c d e f g h i

Resulta: S = abc + def + ghi + adg + beh + cfi = 100 (a + c + f + b + a + d + g) + 10(d + e + f + b + e + h) + (g + h + i + c + f + i) = 200 a + 110b + 101c + 110d + 20e + 11f + 101g + 11h + 2i Mdulo 9 tenemos: S = 2(a + b + c+....h + i) = 2.45 = 0

Como 2001 no es mltiplo de 9, no habr ninguna distribucin para la que la suma indicada tome el valor 2001. 5.- ABCD es un cuadriltero inscrito en una circunferencia de radio 1 de modo que AB es un dimetro y el cuadriltero admite circunferencia inscrita. Probar que: CD 2 5 4 D p C Solucin: Sea O el centro de la semicircunferencia. Pongamos a = BC; b = AD; p = CD; b a 2 = BOD; 2 = AOD; 2 = COD. 2 La condicin necesaria y suficiente para que ABCD admita una circunferencia inscrita es: A p + 2 = a + b (1) O Como 2 + 2 + 2 = 180, entonces

= 90 ( + )
y adems:
a = 2sen; p = 2sen; b = 2sen = 2 cos( + ) = 2 cos cos 2sen sen

Vamos a expresar la condicin (1) en funcin del ngulo y el dato p que determina por completo el cuadriltero. 4 p2 p2 = cos = 1 , 4 2 de donde:

b = 4 p 2 cos psen
sustituyendo en (1), queda:
p + 2 = 2sen + 4 p 2 cos psen

o lo que es lo mismo: 4 p 2 cos + (2 p )sen = p + 2 (2) Por tanto, existir circunferencia inscrita para los valores de p que hagan compatible la ecuacin (2) en la incgnita . Puede expresarse el seno en funcin del coseno y estudiar el discriminante de la ecuacin de segundo grado que se obtiene, pero es ms rpido interpretar la ecuacin (2) como el producto r r escalar de los vectores u (cos , sen ) de mdulo 1 y v 4 p 2 , 2 p . La condicin (2) queda: r v cos = p + 2 (3) r r siendo el ngulo formado por los vectores u y v . r 2 Para que (3) sea compatible debe cumplirse p + 2 v = 4 p 2 + (2 p ) , elevando al cuadrado

y operando queda:
p2 + 8p 4 0

Las races de la ecuacin son p = 2 5 4 .

Como p es positivo la condicin final es: 0 p2 5 4 6.- Determinar la funcin f : N N (siendo N = {1,2,3,...} el conjunto de los nmeros naturales) que cumple, para cualesquiera s, n N, las dos siguientes condiciones: a) f (1) = 1, f (2s) = 1. b) Si n < 2s, entonces f (2s + n) = f (n) + 1. Calcular el valor mximo de f (n) cuando n 2001. Hallar el menor nmero natural n tal que f (n) = 2001.

Solucin Para cada nmero natural n definimos f (n) como la suma de las cifras de la expresin de n escrito en base 2. Est claro que esta funcin f cumple las condiciones a) y b). Adems, es la nica funcin que las cumple, porque el valor de f (n) viene determinado por las condiciones a) y b). Probamos esa afirmacin por induccin sobre n. Si n = 1 o n = 2s, f (n) = 1. Supongamos n > 1, n 2s y que es conocido f (m) para todo m < n; se puede escribir n = 2s + m con m < 2s tomando 2s la mayor potencia de 2 que es menor que n; entonces f (n) = f (m) + 1. Ahora, es fcil resolver las dos cuestiones que nos plantean: En el primer caso, se trata de ver cuntos unos puede tener como mximo un nmero menor o igual que 2001 escrito en base 2. Ese nmero, escrito en base 2, es, obviamente, 1111111111, que corresponde a n = 1023 = 210 - 1. Es f (n) = 10. En el segundo caso, razonando de manera anloga, se observa que la respuesta es n = 2 2001 -1.

RESPUESTAS DE LA XXXVIII OLIMPIADA ESPAOLA

Problema 1
Hallar todos los polinomios P(t) de una variable, que cumplen:
P ( x 2 y 2 ) = P ( x + y ) P ( x y )

para todos los nmeros reales x e y. La ecuacin funcional dada P ( x 2 y 2 ) = P( x + y ) P( x y ) (*) es equivalente a la ecuacin funcional P (uv) = P (u ) P(v) (**) con el cambio de variables u = x + y y v = x y, para todos u, v R. Poniendo u = v = 0 en (**) se obtiene P (0) = ( P(0)) 2 , de donde P(0) = 1 P(0) = 0. Sea P (0) = 1, haciendo v = 0 en (*) se deduce que P (0) = P(u ) P(0) para todo u R, es decir P(u ) 1. Sea ahora P (0) = 0. Entonces P (u ) = uQ(u ), siendo Q(u ) un polinomio de grado una unidad inferior al grado de P(u ). Fcilmente se comprueba que Q(u ) satisface la ecuacin funcional (**). Por tanto P (u ) = u n con n N . Recprocamente se comprueba sin dificultad que P( x) 1 y P( x) = x n con n N satisfacen la ecuacin funcional inicial (*). Tambin puede hacerse sin el cambio de variable haciendo x = y = 0 se llega a P (0) = ( P (0)) 2 . Adems est la solucin trivial P(x) 0.

Problema 2
En un tringulo ABC, A es el pie de la altura relativa al vrtice A y H el ortocentro. AA ' a) Dado un nmero real positivo k tal que = k , encontrar la relacin entre los ngulos B y HA ' C en funcin de k. b) Si B y C son fijos, hallar el lugar geomtrico del vrtice A para cada valor de k.
A C' H B'

a) Tenemos:
BA ' = c cos B ; tg HBA ' = ctg C =

HA ' , AA ' = c sen B . BA '

De donde:
k=
B A' C

AA ' c sen B = tg Btg C = k (1) HA ' c cos B ctg C

b) Poniendo a = BC, tomando unos ejes con origen en el punto medio de BC y eje OX sobre el lado a a BC, resulta B , 0 ; C , 0 y llamando A(x, y), la condicin (1) se escribe: 2 2 a2 y y 2 = k y = k x2 a a 4 x +x 2 2

que, una vez operada resulta:


x2 y2 + = 1 (2) a 2 ka 2 4 4 ecuacin de una elipse en la que distinguimos dos casos: a a 1 k y semieje mayor = 2 2 a a Si k >1, elipse con eje mayor sobre OY, semidistancia focal = k 1 y semieje mayor = n. 2 2

Si k < 1, elipse con eje mayor sobre OX, semidistancia focal =

A F

F'

A'

A'

F'

Problema 3
La funcin g se define sobre los nmeros naturales y satisface las condiciones:
g(2) = 1 g(2n) = g(n) g(2n + 1) = g(2n) + 1

Sea n un nmero natural tal que 1 n 2002. Calcula el valor mximo M de g(n). Calcula tambin cuntos valores de n satisfacen g(n) = M. Para cualquier natural n, consideramos su representacin binaria,
n = ak 2k + ak 1 2k 1 + K + a1 2 + a0 = ak K a1a0( 2) ,

donde aj = 0 o 1. Probaremos por induccin que g ( n ) = a j por induccin sobre k: Para k = 0 es cierto: g 1( 2) = g (1) = 1 . Supuesto cierto para k, hay dos casos para k + 1:
g ak K a1a0 0( 2) = g 2ak K a1a0( 2) = a j ,
j =0 k

( )

j =0

( (

) (

g ak K a1a01( 2) = g 1 + 2ak K a1a0( 2) = 1 + a j


j =0

) (

donde se han aplicado las propiedades de g y la hiptesis inductiva. Entonces g(n) es el nmero de unos de n escrito en base 2.

Como 211 = 2048 > 2002 > 1024 = 210, resulta M = 10. Hay cinco soluciones de g(n) = 10: 1023, 1535, 1791, 1919 y 1983.

Problema 4
Sea n un nmero natural y m el que resulta al escribir en orden inverso las cifras de n. Determinar, si existen, los nmeros de tres cifras que cumplen 2m + S = n, siendo S la suma de las cifras de n.
n = abc = c + 10b + 100a; m = cba = 100c + 10b + a 2 m + S = n nos da: 200c + 20b + 2a + (a + b + c) = 100a + 10b + c, es decir 200c + 11b 97a = 0. Por lo tanto, 200c 97a es mltiplo de 11. Mdulo 11: 2(c + a) es 0, y como mcd (2,11) = 1, resulta que a + c es congruente con 0 mdulo 11. Mdulo 9: 2(c + a + b) congruente con 0, y c + a + b congruente con 0. Por la primera congruencia, c + a = 0, o bien c + a = 11. Si c + a = 0, entonces a = c = 0 y no hay solucin por ser nmeros de tres cifras. Si c + a = 11, entonces b = 7. Por lo tanto, 200c 97a es mltiplo de 7. Trabajando mdulo 7: 4c + a es congruente con 0 mdulo 7, es decir;

4c + a = 0, 7, 14, 21, 28, 35, 42. Como a + c = 11, tenemos que 3c debe tomar uno de los valores -11, -4, 3, 10, 17, 24, o 31 y ser mltiplo de 3. Luego c = 1 o c = 8. Si c = 1, entonces a = 10, imposible. Si c = 8, a = 3. Pero n = 378 no es solucin y no existen nmeros con las condiciones pedidas.

Problema 5
Se consideran 2002 segmentos en el plano tales que la suma de sus longitudes es la unidad. Probar que existe una recta r tal que la suma de las longitudes de las proyecciones de los 2002 segmentos 2 dados sobre r es menor que . 3 Cada segmento determina dos vectores de igual mdulo y sentido opuesto. Consideramos los 2 x 2002 = 4004 vectores as obtenidos y los ordenamos por sus direcciones entre 0 y 2 respecto de un sistema de referencia ortonormal arbitrario. Construimos ahora un polgono convexo de 4004 lados uniendo los vectores uno a continuacin del otro, a partir de uno cualquiera dado. Claramente el permetro de este polgono es 2 Adems es un polgono centrado y simtrico, respecto de un punto O (la prueba de esta observacin es sencilla y es necesario hacerla). Tomamos entonces uno de los lados ms prximos a O; sea d el segmento perpendicular a ese lado

y a su opuesto que pasa por el centro O. La proyeccin del polgono sobre la recta que contiene a este segmento es d y por tanto la suma de las proyecciones sobre la recta anterior es tambin d. Por d otra parte la circunferencia de centro O y radio est totalmente contenida en el interior del 2 polgono y entonces su circunferencia es menor que el permetro del polgono. 2 2 Es decir: d < 2 y d < < . 3 Falta considerar el caso trivial de que todos los segmentos tengan la misma direccin en cuyo caso ni hay polgono pero tomando la recta perpendicular a la direccin comn sale d = 0.

Problema 6
En un polgono regular H de 6n + 1 lados (n entero positivo), r vrtices se pintan de rojo y el resto de azul. Demostrar que el nmero de tringulos issceles que tienen sus tres vrtices del mismo color no depende del modo de distribuir los colores en los vrtices de H. Debido a que el nmero de lados del polgono H deja de resto uno al dividirse entre seis, cada diagonal y cada lado del mismo pertenece slo (exactamente) a tres tringulos issceles distintos (la demostracin es sencilla y se debe hacer). Denotamos por AA, AR y RR los nmeros de segmentos que son lados y diagonales cuyos extremos respectivamente estn coloreados ambos de azul, de azul y de rojo o ambos de rojo. Anlogamente denotamos por AAA, AAR, ARR y RRR el nmero de tringulos issceles cuyos vrtices son los tres azules, dos azules y uno rojo, uno azul y el otro rojo o los tres rojos y ninguno azul, respectivamente. Entonces 3 AA = 3 AAA + AAR, porque cada diagonal o lado de H pertenece a tres tringulos issceles y los tringulos issceles con tres vrtices azules tienen tres lados con sus dos extremos azules. Los tringulos issceles con dos vrtices azules tienen slo un lado con sus extremos de color azul y los tringulos issceles con menos de dos vrtices azules no tiene ningn lado con los extremos del mismo color azul. Anlogamente establecemos: 3 RA = 2 AAR + 2 ARR y 3 RR = ARR + 3 RRR. (se deben probar estas dos nuevas relaciones). Las tres relaciones obtenidas conducen a que: 1 1 1 1 AAA + RRR = RR + AA RA = R ( R 1) + A ( A 1) R A, 2 2 2 2 donde A es el nmero de vrtices azules, A = 6n + 1 R. Esto completa la prueba. Se observa que el resultado es tambin cierto si el polgono H tiene 6n + 5 lados.

RESPUESTAS DE LA XIL OLIMPIADA ESPAOLA

Problema 1
Probar que para cualquier primo p distinto de 2 y 5 existe un mltiplo de p cuyas cifras son todas nueves. Por ejemplo si p = 13, 999999 = 1376923 Solucin de Luis Hernndez Corbato de Madrid.
i 6 7 8 Sea ai el nmero compuesto por i nueves ai = 99K 9 . Supongamos que p tal que | ai i para probar por contradiccin el enunciado. p/

Considrense en dicho caso los nmeros {a1 , a2 ,K a p } , en este conjunto sabemos que no hay ningn ai 0 ( p ) (por hiptesis) . Por tanto al haber p nmeros y slo p 1 restos posibles mdulo
p, se sabe que existen m, n tales que am an 0 ( p ) . Suponemos sin prdida de generalidad que m > n y: m n mn n 6 7 8 6 7 8 6 7 86 7 8 p | am an = 99K 9 99K 9 = 99K 9 00K 0 = am n 10n

Como p 2 y p 5 p / | 10n = 2n 5n p | am n y como am n pertenece al conjunto escogido por ser m n < n y m n 1 se ha llegado a una contradiccin. Por ende:

p ai tal que p | ai
y el enunciado queda probado.

Problema 2
Existe algn conjunto finito de nmeros reales M que contenga al menos dos elementos distintos y que cumpla la propiedad de que para dos nmeros a, b cualesquiera de M, el nmero 2a - b2 sea tambin un elemento de M? Solucin de Vctor Gonzlez Alonso de Burgos. Como M es finito, necesariamente estar acotado. Pongamos M [ x, y ] , con x = Mn M e y = Mx M. Supongamos x 0: Tenemos x 0 2x x 2x k2 < x (k cualquier nmero de M). Esto contradice que x sea el mnimo de M. Por tanto x > 0 y 0 < x < y. En cualquier casi debe ser: (1) x 2x y2 y y adems (2) x 2y y2 y . De (1) se desprende que : x 2x y2 0 x y2 y2 x <y ; que slo se cumple si y (0, 1). De (2) obtenemos que: 2y y2 y y y2 0 y y2; y A esto slo es cierto si y [1, + ) . Como (1) y (2) deben cumplirse a la vez, no existe ningn y R que pueda ser mximo de M por lo que no estara acotado y no sera finito.
90
C A' C' H

Problema 3
Las alturas del tringulo ABC se cortan en el punto H. Se sabe que AB = CH. Determinar el valor del ngulo BCA. Solucin de Ibn Arregui Bilbao del Pas Vasco. ngulo C < 90. Llamaremos A al punto en que la altura de A corta al lado BC del tringulo ABC, y C al punto donde la altura de C corta al lado AB del tringulo ABC. El ngulo CHA es igual al ngulo AHC. En el tringulo CAH, el ngulo CAH es recto, por tanto el ngulo HCA es 90 - . En el tringulo AHC el ngulo HCA es recto, por tanto el ngulo HAC es 90 - . El ngulo HAC es igual al ngulo AAB del tringulo AAB que es rectngulo por tanto el ngulo ABA es . De aqu concluimos que los tringulos CHA y AAB son semejantes, y como CH = AB , son tringulos iguales de donde obtenemos que AA = CA, por tanto el valor de tg C = 1, y C = 45.
A

A'

C'

90

90

B'

ngulo C > 90. Procediendo de modo anlogo el ngulo ACH es igual al ngulo CCB . En el tringulo CCB el ngulo CAH es recto, por tanto el ngulo AHC es 90 - y en el tringulo B CCB el ngulo CCB es recto y por tanto CBC es 90 - . El tringulo AAB es rectngulo en A y por ello BAA es . Entonces los tringulos AAB y ACH son semejantes y tienen la hipotenusa igual, luego son iguales y deducimos AA = AC , entonces la tangente de C vale 1 y C = 135. Finalmente, si fuese C = 90, C coincide con H y CH = 0. Como AB 0, este valor de C no es vlido.

Problema 4
Sea x un nmero real tal que x3 + 2x2 +10x = 20. Demostrar que tanto x como x2 son irracionales.

Solucin: Primero veamos que x no puede ser entero. Esto puede hacerse teniendo en cuenta que si lo fuese, sera un divisor de 20, y basta probar los 8 divisores para comprobar que ninguno verifica la ecuacin. Otro modo de verlo es comprobar que f(x) = x3 + 2x2 +10x 20 es estrictamente creciente (su derivada es positiva para todo x) y adems f(1) = 13 y f(2) = 36. Luego no hay races enteras. Veamos que x no puede ser racional por reduccin al absurdo. Supongamos que x = p/q con q 1 y p/q irreducible. Entonces p 3 = 20q 3 10q 2 p 2qp 2 = q ( 20q 2 10qp 2 p 2 )

Si q fuera estrictamente mayor que 1, la igualdad anterior estara en contradiccin con la hiptesis de que p/q es irreducible. Por tanto q = 1, x sera entero lo que es imposible. Luego x es irracional. Para la irracionalidad de x2 basta ver que 20 2 x 2 x ( x 2 + 10 ) = 20 2 x 2 x = 2 , x + 10 y si x2 fuese racional, tambin los sera x en contra de lo probado.

Problema 5
Cules son las posibles reas de un hexgono con todos los ngulos iguales y cuyos lados miden 1, 2, 3, 4, 5 y 6 en algn orden? Solucin: La idea es prolongar los lados para formar un tringulo equiltero.
a + b + c + d + e + f = 21 l=a+b+c=c+d+e=e+f+a 3l =21 + a + c + e, por tanto l = 7 + (a + c + e) / 3

El valor ms pequeo de a + c + e es 6 y el ms grande 15 as que 9 l 12

a Si a + c + e = 6, entonces son: (a, c, e) = (1, 2, 3) y (b, c, d) = (4, 5, 6) a Si a + c + e = 9 el nico caso posible es: (a ,c, e) = (1, 3, 5) y (b, c, d) = (2, 4, 6) Si a + c + e = 12 el nico caso posible es (a ,c, e) = (2, 4, 6) Si a + c + e = 15 el nico posible es (4, 5, 6).

c c

Como el rea del tringulo de lado l es l 2 reas posibles son: Si a + c + e = 6, entonces l = 9 y el rea Si a + c + e = 9, entonces l Si a + c + e = 4, entonces l Si a + c + e = 5, entonces l

3 3 2 y la del hexgono es l - (a 2 + c 2 + e 2 ) , las 4 4

67 3 4 65 3 = 10 y el rea 4 65 3 =11 y el rea 4 67 3 = 12 y el rea 4

Problema 6
Ensartamos 2n bolas blancas y 2n bolas formando una cadena abierta. Demuestra que, se haga en el orden que se haga, siempre es posible cortar un segmento de cadena exactamente con n bolas blancas y n bolas negras.

Solucin de Mohammed Blanca Ruiz de Valencia. Tenemos la cadena con el total de 4n bolas, 2n blancas y 2n negras. Cogemos un grupo de un extremos con 2n bolas, este grupo tendr x bolas negras e y bolas blancas, de forma que la diferencia es x y = 2k para k {n,1 n,K , 0,K n 1, n} . Vamos movindonos de una en una posicin hacia el extremo contrario, en cada movimiento la diferencia vara en 2 o no vara, es decir k aumente en 1, disminuye en 1 o no cambia. La diferencia vara en 2 si la bola que se deja y que se coge son de distinto color y no se mantiene si son del mismo color. La posicin final, es decir en el otro extremo, tendr las bolas al revs, x bolas blancas e y bolas negras con lo que la diferencia (blancas - negras) ser ahora y x = 2k, para el mismo k. Es decir que k pasa de una posicin a su opuesta con el mismo valor absoluto. Como k slo puede variar de 1 en 1 tiene que pasar por el cero ya que no se lo puede saltar. En el momento en que k = 0, x = y = n, c.q.d. Siempre se podr cortar un segmento de longitud 2n con n bolas blancas y n bolas negras.

RESPUESTAS DE LA XL OLIMPIADA ESPAOLA


Problema 1
Tenemos un conjunto de 221 nmeros reales e cuya suma es 110721. Los disponemos formando un rectngulo de modo que todas las filas y la primera y ltima columna son progresiones aritmticas de ms de un elemento. Probar que la suma de los elementos de las cuatro esquinas vale 2004 Solucin Denotaremos por aij al elemento de la fila i-sima y columna j-sima del rectngulo Pongamos n para el nmero de filas, m para el de columnas y S para la suma de los nm elementos. Con notacin matricial queda:
1 a1 1 a M = 2 L a1 n

a12 L a1m 2 m a2 L a2 L L L 2 m L an an

Sumando por filas y llamando Sk a la suma de la fila k, resulta: a1 + a m S1 = 1 1 m 2 1 a + am S 2 = 2 2 m 2 .......................


Sn = y sumando miembro a miembro queda: S = S1 + S2 + K + Sn = m nm 1 1 1 1 m m m ( a1 + a1 ( a1 + an + a1m + anm ) 2 + K + an ) + ( a1 + a2 + K + an ) = 2 4
1 m m a1 + a1 n + a1 + an =

m a1 n + an m 2

4 S 4110721 = = 2004 nm 221

Problema 2 ABCD es un cuadriltero cualquiera, P y Q los puntos medios de las diagonales BD y AC respectivamente. Las paralelas por P y Q a la otro diagonal se cortan en O. Si unimos O con las cuatro puntos medios de los lados X, Y, T se forman cuatro cuadrilteros, OXBY, OYCZ, OZDT y OTAX. Probar que los cuatro cuadrilteros tienen la misma rea.

A X P B Q Y O Z D T

Zy

Solucin 1 (oficial).
Bastar probar que el rea de cada cuadriltero es la cuarta parte del rea total. La quebrada APC divide al cuadriltero en dos partes de igual rea pues AP es la mediana de ABD y PC lo es de CBD. La quebrada TPZ divide al cuadriltero APCB (sombreado) en dos partes de igual rea pues PT es mediana de APD y PZ es mediana de CPD. Tenemos ya probado que el rea del cuadriltero TPZD es la A cuarta parte del rea del cuadriltero inicial. T X Finalmente TZ es paralela a OP por serlo ambas a AC; luego los P D tringulos TPZ y TOZ tienen la misma rea y lo mismo les ocurre B a los cuadrilteros TPZD y TOZD. Del mismo modo se probara para los otros tres cuadrilteros. O
Q Y Z

Solucin 2 (de la concursante Elisa Garca Lorenzo) La frmula de la superficie del cuadriltero A AC BDsen S= 2 X AC al ser ZT la paralela Adems ZT = XY = 2 T' media del tringulo ACD y XY la paralela Q B media del tringulo ABC. BD T Igualmente: XT = ZY = 2 O Y Para probar el enunciado bastar probar que:
P C Z' Z D

es:

AC BD XT AO sen = 4 sen 2 2 AC BDsen = 2 XT AOsen ACsen = 2 AOsen AQsen = AOsen AQ AO = sen sen que es el teorema del seno en el tringulo AQO. Queda probado el enunciado por extensin de la demostracin a los 4 cuadrilteros pequeos que resultan ser una cuarta parte de grande.

Solucin 3 (de Marco Castrilln Lpez).


A

Q B O Y P

Al ser OP paralela a AC, los tringulos OXY, PXY tienen la misma base e igual altura y por tanto la misma rea. De ah que los cuadrilteros OXBY, PXBY tambin tienen la misma rea, pero el rea de PXBY (en amarillo en la figura) es la cuarta parte del cuadriltero inicial al ser semejantes con razn 2 del grande al pequeo.

Problema 3 Se representa por Z el conjunto de todos los enteros. Hallar todas las funciones f : Z Z , tales que para cualesquiera x, y enteros se verifica:
f ( x + f ( y )) = f ( x) y.

Solucin:
Primeramente observemos que f ( x + nf ( y )) = f ( x) ny. Para n = 0 es obvio, y por induccin suponiendo que para cada entero n 1

f ( x + (n 1) f ( y )) = f ( x) (n 1) y,
entonces: f ( x + n f ( y )) = f ( x + (n 1) f ( y ) + f ( y )) = f ( x + (n 1) f ( y )) y = = f ( x) (n 1) y y = f ( x) ny. Anlogamente se prueba para cada entero n 1. Por tanto

f (1 + f (1) f (1)) = 0.
Poniendo k = 1 + f (1) f (1) = 1 + f (1) 2 > 0, se tiene f ( x) = f ( x + f (k )) = f ( x) k , que es una contradiccin. Deducimos que no existen funciones que satisfagan la condicin requerida.

Problema 4 Existe alguna potencia de 2 que al escribirla en el sistema decimal tenga todos sus dgitos distintos de cero y sea posible reordenar los mismos para formar con ellos otra potencia de 2 ?. Justificar la respuesta

Solucin 1 (oficial): Supongamos que exista tal potencia de 2, es decir, que haya dos potencias de 2 cuyas expresiones decimales slo difieran en el orden de colocacin de los dgitos. Claramente ninguna de las dos potencias es divisible por 3, y ambas dejan el mismo resto cuando se dividen por 9. Esto ltimo se debe a que el resto de un nmero al dividirse por 9 es congruente, mdulo 9, con la suma de sus dgitos. Por otra parte la mayor de ambas potencias se obtiene de la menor multiplicando sta por 2, 4 u 8 (de otra manera no tendran ambas el mismo nmero de dgitos). Sin embargo al multiplicar la menor potencia de las dos por 2, 4 u 8, cambia el resto cuando se divide por 9. Los restos de las sucesivas potencias de 2 al dividirse por 9 forman una sucesin peridica. Efectivamente, los restos de: 2, 4,8,16,32, 64,128, 256,512,1024, 2048, 4096,..., son: 2, 4,8, 7,5,1, 2, 4,8, 7,5,1,... Esta sucesin tiene periodo 6, porque para todo n entero positivo
2n + 6 2n = 2n (26 20 ) = 63 2n , y este nmero es divisible por 9, por lo que ambas potencias dejan el mismo resto. No es posible por tanto, reordenar los dgitos de una potencia de 2 para obtener otra potencia distinta de 2.

Solucin 2 (del concursante Lander Ramos Garrido). No existe ningn nmero que cumpla las condiciones del enunciado. En primer lugar, ambos deben tener las mismas cifras lo que implica que el nmero de cifras sea el mismo, as que el cociente entre ambos no debe ser mayor que 8, porque si fuera 16 se alterara el nmero de cifras. Otra condicin que han de cumplir es, obviamente, que la suma de sus cifras sea la misma. Como duplicar un nmero implica que la nueva suma de sus cifras sea el doble de la antigua menos 9x, donde x es el nmero de llevadas ya que a cada llevada restas 10 a un nmero y sumas 1 al siguiente. Para que la suma fuera igual tiene que cumplirse 2 y 9x = 0 siendo y la suma de las cifras antiguas. Entonces y debe ser mltiplo de 9 para que se cumpla la ecuacin anterior ya que en caso contrario habra medias llevadas, absurdo. Si y es mltiplo de 9, segn el criterio de divisibilidad, el nmero tambin debera ser mltiplo de 9, pero como estamos tratando potencias de 2, no habr ningn nmero que cumpla esas caractersticas. Para 4 y 8 el proceso es parecido, las frmulas seran: Para 4: (z son las llevadas en la segunda duplicacin). 2 ( 2 x 9 y ) 9 x = 0 4 x 18 y 9 z = 0 4 x = 9 ( 2 y + z )
Tampoco podra ser ya que 2y + z es un natural. Para 8: (a son las llevadas en la tercera duplicacin). 2 4 x 9 ( 2 y + z ) 9a = 0 8 x = 9 ( 4 y + 8 z + a ) . Y tampoco podra ser.

Problema 5 Demostrar que la condicin necesaria y suficiente para que, en el tringulo ABC, la mediana desde B sea dividida en tras partes iguales por la circunferencia inscrita en el tringulo, es
a b c = = 5 10 13

Solucin. a) la condicin es necesaria. Sea ABC un tringulo tal que la mediana BK (K punto medio de AC) corte a la circunferencia inscrita en dos puntos, M y N, tales que
BM = MN = NK = x

Sea T el punto de tangencia del crculo inscrito con el lado BC.


C a T I M B c A N b K

Las siguientes relaciones se verifican en cualquier tringulo: a + c b = 2 BT 2a 2 + 2c 2 b 2 = 4 BK 2 (La primera se deduce sin ms de BT + CT = a, BT CT = c b; la segunda frmula de Apolonio o de la mediana- se puede tambin obtener completando el tringulo ABC hasta obtener un paralelogramo ABCD). Entonces resulta 2a 2 + 2c 2 b 2 = 36 x 2 (1) La potencia del vrtice B respecto del crculo inscrito se puede escribir de dos maneras: BT 2 = BM BN , con lo cual

( a + c b)

= 8 x 2 (2)

Como, evidentemente, en el tringulo del problema, los puntos B y K estn igualmente alejados del centro del crculo inscrito, resulta BC = KC, de donde
b = 2a

Sustituyendo esta ltima igualdad en (1) y (2), obtenemos c 2 a 2 = 18 x 2 , y ya que c a 0, x 0, resulta c+a 9 c 13 = , de donde = ca 4 a 5 Por lo tanto, a b c = = . 5 10 13 b) la condicin es suficiente.

(c a)

= 8x2

No hay prdida de la generalidad en suponer que a = 5, b = 10, c = 13. Sustituyendo los valores de los lados en las frmulas utilizadas en la parte a), resulta BK = 6 2, BT 2 = 16 = BM BN y en el inradio r=
C a T I M B H c A N b K

S = 6 14 p

(calculando S por la frmula de Hern). El tringulo BCK es issceles, as que la bisectriz del ngulo C es tambin altura. Sea H = CI BK ; consideremos el tringulo rectngulo BIT; entonces 22 47 BI = 4 + r = 14
2 2 2

4 , y finalmente en IHM, HM 2 = r 2 HI 2 = 2. 7 Como H es el punto medio de MN, resulta MN = 2 2 , luego la mediana BK queda, en efecto, dividida en tres partes iguales por el crculo inscrito. por otra parte, en BIH, HI 2 =

Problema 6 Colocamos, formando una circunferencia, 2004 fichas bicolores: blancas por una cara y negras por la otra. Un movimiento consiste en elegir una ficha negra, y dar la vuelta a tres fichas: la elegida, la de su derecha y la de su izquierda. Supongamos que inicialmente hay una sola ficha con la cara negra hacia arriba. Ser posible, repitiendo el movimiento descrito, conseguir que todas las fichas tengan la cara blanca hacia arriba? Y si tuviramos 2003 fichas, entre las cuales exactamente una tiene al comienzo la cara negra hacia arriba?
Solucin Numeremos las fichas desde 1 hasta 2004: la 1 es negra y las restantes son blancas. Cada ficha inicialmente blanca debe ser tocada un nmero par de veces, para que al final del proceso siga teniendo la cara blanca hacia arriba. Cada movimiento posible cambia el nmero de fichas negras en un nmero impar: BNB pasa a NBN : el nmero de fichas negras aumenta en 1 NNB pasa a BBN : el nmero de fichas negras disminuye en 1 BNN pasa a NBB: el nmero de fichas negras disminuye en 1 NNN pasa a BBB: el nmero de fichas negras disminuye en 3 Como inicialmente hay exactamente una ficha negra, el nmero total de movimientos para tener las 2004 fichas con la cara blanca hacia arriba debe se impar. Designamos por xi el nmero de movimientos realizados eligiendo la ficha i (que debe ser negra). La ficha que ocupa el lugar i cambia de color en los movimientos en que la elegimos a ella (xi), a la de su izquierda (xi-1) o a la de su derecha (xi+1). Por lo tanto, (xi-1 + xi + xi + 1) es el nmero de veces que hemos dado la vuelta a la ficha que ocupa el lugar i (2004+1 se identifica con 1, y 2003 +2 se identifica con 1) El nmero total de movimientos ser:
N = ( x1 + x2 + x3 ) + x4 + K + ( x2002 + x2003 + x2004 )

Como 2004 es mltiplo de 3, N es la suma del nmero de veces que hemos dado la vuelta a las fichas en los lugares 2, 5 ... 3k+2, ... 2003, todas ellas blancas al principio: as que N, suma de nmeros pares, debera ser par: contradiccin, pues N es impar. Por lo tanto, no ser posible conseguir que las 2004 fichas tengan la cara blanca hacia arriba. Con 2003 fichas si es posible: iniciando el movimiento sobre la ficha 1, (nica negra al principio), y repitindolo sobre las fichas que ocupan los lugares 2 .....2001,2002 llegaramos a la configuracin NNN NNN .... NNN BB Eligiendo ahora las fichas que ocupan los lugares 2, 5 ... 3k+2.... 2000 tendramos: BBB BBB .......... BBB BB en la que todas las fichas tendran la cara blanca hacia arriba.

You might also like